MedSurge Exam 1 Prep

Pataasin ang iyong marka sa homework at exams ngayon gamit ang Quizwiz!

A nurse observes a window washer falling 25 feet to the ground. The nurse rushes to the scene and determines that the person is in cardiopulmonary arrest. What should the nurse do first? 1. Feel for a pulse 2. Begin chest compressions 3. leave to call for assistance 4. Perform the abdominal thrust maneuver

2. Begin chest compressions According to the 2010 American Heart Association Guidelines for CPR, the first step is to feel for a pulse. In this case, it has been established the patient has no pulse, therefore, chest compressions are initiated. Do not leave the patient to call for assistance. The abdominal thrust (Heimlich) maneuver is used to relieve airway obstruction and is not appropriate in this instance.

A client reports a history of bilateral blanching and pain in the fingers on exposure to cold. When rewarmed, the fingers become bright red and "tingly" with a slow return to their usual color. The client smokes one to two packs of cigarettes per day. The nurse determines that the client has Raynaud's disease and not Raynaud's phenomenon because of the: 1. Tingling sensation 2. Skin color changes 3. Bilateral involvement 4. Changes in skin temperature

3. Bilateral involvement Raynaud - a condition in which some areas of the body feel numb and cool in certain circumstances. Raynaud's phenomenon has unilateral involvement, whereas Raynaud's disease has bilateral involvement. Tingling sensation indicates return of blood flow and is characteristic of both Raynaud's phenomenon and Raynaud's disease. Skin color changes indicate blood return and are characteristic of both Raynaud's phenomenon and Raynaud's disease. Changes in skin temperature indicate lack of blood supply and are characteristic of both Raynaud's phenomenon and Raynaud's disease.

The nurse is completing an assessment on a couple seeking genetic counseling for sickle cell anemia. Both prospective parents carry sickle cell traits. The nurse recognizes that the couple has what chance of having a child who develops the disease? 1. 25% 2. 50% 3. 75% 4. 100%

1. 25% Sickle cell is an autosomal recessive genetic disorder. If both individuals have sickle cell traits, there is a 25% chance they will produce a child with the disease. Other options, such as 50%, 75%, and 100%, are not plausible.

The nurse is conducting a nutrition class for a group of clients with congestive heart failure (CHF). It would be most important for the nurse to explain the importance of: 1. Restricting fluid intake. 2. Choosing fresh or frozen vegetables instead of canned ones. 3. Eating a low caloric diet to reduce weight. 4. Recognizing which products are high in cholesterol.

2. Choosing fresh or frozen vegetables instead of canned ones. The key principle to teach CHF clients is the importance of decreasing sodium in their diet and which foods contain sodium. If sodium is decreased, water retention will decrease also. Fresh or frozen vegetables have less sodium than canned ones. If the client is on a low sodium diet and receiving diuretics but continues to be fluid overloaded, then fluid restriction may be instituted. A low caloric diet is not indicated for all CHF clients. Some are very thin because of various factors, including the work of breathing and rapid heart rate. A low cholesterol diet is important for clients with coronary artery disease and for the American population in general, but is not specifically related to CHF.

What is the most effective way for the nurse to loosen respiratory secretions for a client with an endotracheal tube? 1. Increase oral fluid intake 2. Provide chest physiotherapy 3. Humidify the prescribed oxygen 4. Instill a saturated solution of potassium iodide

3. Humidify the prescribed oxygen ecause the client has an endotracheal tube in place, secretions can be loosened by administration of humidified oxygen and by frequent turning. A client with an endotracheal tube in place is not permitted fluids by mouth. Providing chest physiotherapy is too vigorous for a client with an endotracheal tube. Potassium is never instilled into the lungs.

A nurse is caring for a client who is receiving aspirin therapy. Which clinical indicator related to this therapy should be a matter of concern to the nurse? 1. Urinary calculi 2. Atrophy of the liver 3. Prolonged bleeding time 4. Premature erythrocyte destruction

3. Prolonged bleeding time Aspirin interferes with platelet aggregation, thereby lengthening bleeding time. Urate excretion is enhanced by high doses of aspirin. Aspirin does not cause atrophy of the liver; it is readily broken down in the gastrointestinal tract and liver. Aspirin does not destroy erythrocytes.

The health care provider prescribes nitroglycerin ointment to be applied topically every eight hours for a client who was admitted for chest pain and a myocardial infarction (MI). Which statement, if made by the client, would indicate understanding of the side effects of nitroglycerin ointment? "I may experience: 1 A headache." 2 Increased blood pressure readings." 3 A slow pulse rate." 4 Confusion."

1 A headache." The most common side effect of nitroglycerin is a headache. Additional cardiovascular side effects are tachycardia, not bradycardia; hypotension, not hypertension; and dizziness, not confusion.

A client has been admitted for an upper respiratory tract infection secondary to chronic obstructive pulmonary disease (COPD). The nurse should expect which findings when auscultating the client's breath sounds? 1.Coarse crackles 2. Prolonged inspiration 3. Short rapid inspiration 4. Normal breath sounds

1.Coarse crackles Coarse crackles and rhonchi most often are auscultated in COPD clients who have had an exacerbation. Clients would exhibit prolonged expiration, not prolonged inspiration. The client would not exhibit short, rapid inspiration. The client would not exhibit normal breath sounds.

A client admitted for uncontrolled hypertension and chest pain was prescribed a low sodium diet and started on furosemide (Lasix). The nurse should instruct the client to include which foods in the diet? 1 Cabbage 2 Liver 3 Apples 4 Bananas

4 Bananas Furosemide is a loop diuretic that eliminates potassium by preventing renal absorption. Bananas have a significant amount of potassium. Bananas: 450 mg; cabbage: 243 mg; liver: 73.6 mg; apples: 100-120 mg.

What is Hodgkin Lymphoma?

Cancer of the part of the immune system called the lymphatic system.

What is Peritoneal dialysis

Peritoneal dialysis (PD) is a treatment for patients with severe chronic kidney disease.

What is hepatomegaly?

abnormal enlargement of the liver

What is hyperglycemia?

an excess of glucose in the bloodstream, often associated with diabetes mellitus.

An African American woman is diagnosed with primary hypertension. She asks, "Is hypertension a disease of African American people?" What is the nurse's best response? 1 "The prevalence of hypertension is about equal for women of all races." 2 "The higher-risk population is composed of African American men and women." 3 "The highest-risk population consists of older Caucasian American men and women." 4 "The prevalence of hypertension is greater for African American women than for African American men."

2 "The higher-risk population is composed of African American men and women." African Americans represent a higher-risk population than Caucasian Americans for hypertension; the reason is unknown. African American women are more frequently affected by hypertension than are Caucasian women. African Americans of both sexes have a higher prevalence than Caucasian Americans of both sexes. African American men have a higher risk than African American women.

While a client with an abdominal aortic aneurysm is being prepared for surgery, the client complains of feeling lightheaded. The client is pale and has a rapid pulse. The nurse concludes that the client is: 1 Hyperventilating. 2 Going into shock. 3 Experiencing anxiety. 4 Developing an infection.

2 Going into shock. The clinical findings are early signs of shock. Shock ensues rapidly after a ruptured aortic aneurysm because of profound hemorrhage. The nurse can observe hyperventilation by watching the client's breathing patterns; rapid respirations are expected with hyperventilation. There are no data that indicate that the client is hyperventilating. Anxiety usually is not associated with lightheadedness unless there is accompanying hyperventilation. The signs and symptoms are not inclusive enough to indicate infection; there is no indication of fever.

A client presenting to the emergency room with chest pain and dizziness was found to be having a myocardial infarction and subsequently suffered cardiac arrest. The emergency room health care team was able to successfully resuscitate the client. Lab work shows that the client now is acidotic. The nurse understands that the acidic serum pH most likely is caused from : 1. fat-forming ketoacids that are broken down 2. the client receiving too much sodium bicarbonate during resuscitation efforts 3. the decreased tissue percussion that subsequently caused lactic acid production 4. an irregular heartbeat the client experienced during cardiac arrest

3. the decreased tissue percussion that subsequently caused lactic acid production Cardiac arrest causes decreased tissue perfusion, which results in ischemia and cardiac insufficiency. Cardiac insufficiency causes anaerobic metabolism, which leads to lactic acid production. Fat-forming ketoacids occur in diabetes. Too much sodium bicarbonate causes alkalosis, not acidosis. An irregular heartbeat does not cause acidosis necessarily.

A client presenting with an acute asthma attack is being assessed in the emergency room. The client's spouse reports that the client currently is being treated for an upper respiratory infection. The nurse should understand that the client most likely has which type of asthma? 1. Allergic 2. Emotional 3. Extrinsic 4. Intrinsic

4. Intrinsic Intrinsic asthma is triggered by an internal factor such as a cold. Intrinsic asthma does not have an identifiable allergen. Asthma related to emotions is considered as extrinsic asthma. Extrinsic asthma includes allergens such as pet dander, dust mites, mold, dust, etc.

A child has been diagnosed with hemophilia type A after experiencing excessive bleeding from a minor trauma. The explanation for how a person gets hemophilia is: 1. Inherited X linked recessive trait 2. Inherited Y linked recessive trait 3. Inherited X linked dominant trait 4. Inherited Y linked dominant trait

1. Inherited X linked recessive trait Hemophilia A is an X linked recessive trait, meaning daughters who have the gene are carriers and sons with the gene have the condition. The trait is not carried on the Y chromosome. Hemophilia is an X linked trait and it is not dominant.

A client who has always been active is diagnosed with atherosclerosis and hypertension. The client is interested in measures that will help promote and maintain health. What recommendation by the nurse will help the client maintain blood vessel potency? 1. Practice relaxation techniques 2. Lead a more sedentary lifestyle 3. Decrease the amount of exercise 4. Increase saturated fats in the diet

1. Practice relaxation techniques Research has shown that decreasing stress will slow the rate of atherosclerotic development. Exercise is thought to decrease atherosclerosis and the formation of lipid plaques. Saturated fats in the diet are contraindicated because they increase the risk for atherosclerosis.

A nurse is providing discharge instructions about digoxin (Lanoxin). Which response should a nurse include as a reason for a client to withhold the digoxin? 1.Chest pain 2. Blurred vision 3. Persistent hiccups 4. Increased urinary output

2. Blurred vision Digoxin (Lanoxin) - Antiarrhythmic and blood pressure support. It can treat heart failure and heart rhythm problems. Visual disturbances, such as blurred or yellow vision, may be evidence of digoxin toxicity. Chest pain is not a toxic effect of digoxin. Persistent hiccups (singultus) are not related to digoxin toxicity. An increased urinary output is not a sign of digoxin toxicity; it may be a sign of a therapeutic response to the drug, an improved cardiac output.

A nurse gave a client naloxone (Narcan). To evaluate the effectiveness of the medication, the nurse should assess for: 1.Change in level of consciousness. 2. Increased pain. 3. Increased respiration. 4. Decreased heart rate

3. Increased respiration. Naloxone is given for decreased respirations caused by opioid overdose. The amount given is determined by the respiratory status, not the level of consciousness. An undesirable side effect of naloxone is pain and rapid heart rate with dysrhythmias.

A health care provider prescribes enalapril (Vasotec) for a client. Which is the most important nursing action? 1. assess the client for hypokalemia 2. Ensure that the medication is ingested with food 3. Monitor the client's blood pressure during therapy 4. Teach that a missed dose can be doubled at the next scheduled time

3. Monitor the client's blood pressure during therapy Enalapril is an antihypertensive. A lowering of the client's blood pressure reflects a therapeutic response and needs to be monitored frequently. The client may be at risk for hyperkalemia, not hypokalemia. Enalapril may be taken without regard to meals. Doubling a dose is unsafe as it may cause an extreme lowering of blood pressure. A missed dose can be taken as long as it is not close to the next scheduled dose.

While conducting an initial assessment on a client, which classic sign would alert the nurse that the client has chronic obstructive pulmonary disease (COPD)? 1. Barrel chest 2. Cyanosis 3 .Hyperventilation 4 . Lordosis

1. Barrel chest Clients with COPD often develop a barrel chest over time because of air being trapped, thus resulting in enlarged lungs and thoracic cavity. This also causes the lungs not to have as much flexibility. Cyanosis is a bluish discoloration, especially of the skin and mucous membranes, caused by excessive concentration of deoxyhemoglobin in the blood caused by deoxygenation. COPD sufferers can exhibit this, but barrel chest is the most obvious sign as other respiratory/cardiovascular disorders can cause cyanosis as well. Hyperventilation is the act of breathing faster or deeper than normal, which causes excessive expulsion of circulating carbon dioxide. This causes the arterial concentration of carbon dioxide (PaCO2) to fall below normal, raising blood pH, and results in alkalosis. COPD sufferers can experience hyperventilation, but barrel chest is the classic sign of COPD. Lordiosis is an unusual inward curving of the spine in the lower part of the back. It can be considered medically significant; however, it is not associated with classic signs of COPD.

A nurse is caring for a client with a history of chronic obstructive pulmonary disease (COPD). What complications are associated most commonly with COPD? 1. Cardiac problems 2. Joint inflammation 3. Kidney dysfunction 4. Peripheral neuropathy

1. Cardiac problems COPD causes increased pressure in the pulmonary circulation. The right side of the heart hypertrophies (cor pulmonale), causing right ventricular heart failure. The skeletal system is not directly related to the pulmonary system; joint inflammation does not occur because of COPD. Kidney dysfunction is not as closely related to the pulmonary system as is the cardiac system; kidney problems usually do not occur because of COPD. Peripheral nerves are not as closely related to the pulmonary system as to the cardiac system; peripheral neuropathy does not occur because of COPD.

A nurse is caring for a client who has chest tubes inserted to treat a hemothorax that resulted from a crushing chest injury. A commonality of the various stationary chest tube drainage systems is that the first chamber is designed to: 1. Collect drainage 2. Ensure adequate suction 3. Maintain negative pressure 4. Sustain a continuance of the water seal

1. Collect drainage The chamber closest to the client in a three-chamber system is the first chamber; it collects drainage. Chamber 2 is the water seal that ensures that air does not enter the pleural space. Chamber 3 is the suction control chamber of the system. The third chamber in a three-chamber system is the suction regulator when it is attached to a source of suction. Chamber 1, the chamber closest to the client in a three-chamber system, does not maintain negative pressure. The second chamber is the water-seal chamber that prevents air from entering the client's pleural space.

The occurrence of which condition would warrant the nurse calling the primary health care provider to discontinue the intravenous (IV) fluids? 1. Crackles in lungs 2. Poor skin turgor 3. Urine output of 240 mL over 8 hours 4. Increase in blood pressure from 110/76 to 130/68 mm Hg

1. Crackles in lungs Crackles in the lungs indicate the client is overloaded with fluids. The nurse should notify the primary health care provider to discontinue the IV fluid. Poor skin turgor is a sign that the client needs fluids. A urine output of 240 mL in 8 hours is adequate per the textbooks, but may not be adequate for certain clients. Therefore, simply having a urine output of 30 cc/hr is not an indication that the IV fluid should be decreased or discontinued. An increase in blood pressure is to be expected with administration of fluid.

A client's respiratory status may be affected after abdominal surgery. The nurse documents the behavioral objective for this client. What statement is a behavioral objective? 1. Demonstrates the technique of coughing and deep breathing. 2. Respirations will improve with coughing and deep breathing. 3. Coughing and deep breathing will facilitate output of secretions. 4. Will cough and deep breathe five or six times every hour while awake.

1. Demonstrates the technique of coughing and deep breathing. Demonstrating the technique of coughing and deep breathing is an objective that includes observable client behavior, which is specified by amount and time and therefore is measurable. The statement that respirations will improve with coughing and deep breathing is an objective not stated in measurable terms. The statement coughing and deep breathing will facilitate output of secretions is not stated in measurable terms. Telling the client to cough and deep breathe five or six times every hour while awake is a statement, not an objective.

When assessing the client with peripheral arterial disease, the nurse anticipates the presence of which clinical manifestations? Select all that apply. 1. Dependent rubor 2. Warm extremities 3. Ulcers on the toes 4. Thick, hardened skin 5. Delayed capillary refill

1. Dependent rubor 3. Ulcers on the toes 5. Delayed capillary refill Peripheral arterial disease affects arterial circulation and results in delayed and impaired circulation to the extremities. As a result, the extremities exhibit rubor while in the dependent position and pallor while elevated, ulcers on the feet and toes, cool skin, and capillary refill greater than three seconds. Warm extremities and thick, hardened skin occur in the presence of venous disease.

A client with heart failure is digitalized and placed on a maintenance dose of digoxin (Lanoxin) 0.25 mg by mouth daily. What responses does the nurse expect the client to exhibit when a therapeutic effect of digoxin is achieved? 1. Diuresis and decreased pulse rate 2. Increased blood pressure and weight loss 3. Regular pulse rhythm and stable fluid balance 4. Corrected heart murmur and decreased pulse pressure

1. Diuresis and decreased pulse rate Digoxin slows the heart rate, which is reflected in a slowing of the pulse; it also increases kidney perfusion, which promotes urine formation, resulting in diuresis and decreased edema. Digoxin will decrease, not increase, the blood pressure; digoxin does promote weight loss through diuresis. Although digoxin produces diuresis as a result of improved cardiac output, which increases fluid output, it does not regulate an irregular pulse. Digoxin will not correct a heart murmur or decrease the pulse pressure.

An 83-year-old client is diagnosed with left-sided congestive heart failure. Which assessment findings should the nurse expect to find on this client? Select all that apply. 1. Dyspnea 2. Crackles 3. Peripheral edema 4. Jugular distention 5. Cool extremities

1. Dyspnea 2. Crackles 5. Cool extremities Left-sided heart failure causes impaired tissue perfusion, pulmonary congestion, and pulmonary edema, which also causes signs and symptoms such as crackles and dyspnea. Decreased cardiac output causes decreased blood flow to major body organs, especially the kidneys. Peripheral edema and jugular distention are signs of right sided congestive heart failure.

Nesiritide (Natrecor), a vasodilator, is prescribed for a client with acute heart failure and pulmonary edema. The nurse is assessing the client's response to the medication. Which clinical manifestation should decrease when the medication is effective? 1. Dyspnea 2. Hypotension 3. Unstable angina 4. Premature heartbeats

1. Dyspnea Human B-type natriuretic peptide binds to receptors in vascular smooth muscle and endothelial cells, leading to smooth muscle relaxation. Dyspnea will decrease as a result of the action of nesiritide. Hypotension will not decrease; it is a side effect of nesiritide. Nesiritide is not used for unstable angina. Nesiritide is not an antidysrhythmic.

A nurse is caring for a client with a pneumothorax who has a chest tube in place. What should the nurse do when caring for this client? 1. Encourage range of motion to the client's arm on the affected side. 2. Administer the prescribed cough suppressant at the prescribed times 3. Empty and measure the drainage in the collection chamber each shift 4. Apply clamps below the insertion site when getting the client out of bed

1. Encourage range of motion to the client's arm on the affected side. Range-of-motion exercises to the client's arm on the affected side promote aeration of the reexpanding lung and maintenance of function in the arm and shoulder. Cough suppressants are not indicated because coughing and deep breathing are encouraged to help reexpand the lung. Drainage is marked with time tapes on the side of the device. The closed system is not entered for emptying; when full, the entire device is replaced. Clamps are not necessary and should be avoided because of the danger of precipitating a tension pneumothorax.

A client returns to the unit fully awake after a bronchoscopy and biopsy. What is the most important nursing intervention? 1. Evaluate the presence of a gag reflex 2. Provide ice chips as a comfort measure 3. encourage the client to cough frequently 4. Advise the client to stay flat for several hours

1. Evaluate the presence of a gag reflex Because of the administration of a local anesthetic during bronchoscopy, fluids and food should be withheld until the gag reflex returns to prevent aspiration. Ice chips must not be given until the gag reflex returns. Coughing should not be encouraged because it might initiate bleeding from the biopsy site. Lying flat will increase the risk for aspiration.

Several days after a client had a total laryngectomy, the health care provider prescribes a progressive diet as tolerated. What should the nurse do? 1. Keep suction apparatus readily available in case excessive respiratory secretions occur 2. administer the diet through a nasogastric tube until the suture line heals 3. Encourage intake of pureed foods to help promote the swallowing reflex 4. Administer the prescribed pain medication before meals to limit discomfort

1. Keep suction apparatus readily available in case excessive respiratory secretions occur Initial attempts at oral feeding may cause a choking feeling that may produce severe coughing that raises secretions. Effective coughing is difficult because with a laryngectomy there is no glottis to close to assist with an expulsive cough. Excessive secretions may block the airway and must be removed by suctioning. Swallowing does not have an adverse effect on the suture line; a nasogastric tube is not used because it can traumatize the suture line. A progressive diet is started with liquids, not pureed foods. Airway patency is the priority, not administering medication for pain.

An 85-year-old client has a serum potassium level of 6.7 mEq/L. Which nursing action is a priority at this time? 1. Monitor for cardiovascular irregularities. 2. Inquire about changes in bowel patterns. 3. Assess client for leg muscle twitching or weakness. 4. Assess client for signs and symptoms of dehydration.

1. Monitor for cardiovascular irregularities. Cardiovascular changes are the most severe problems of hyperkalemia and are the most common cause of death from hyperkalemia. Changes in bowel patterns and leg muscle twitching and weakness are signs of hyperkalemia, but are not life threatening. Dehydration may be a cause of hyperkalemia.

A client with heart failure is receiving digoxin (Lanoxin) and hydrochlorothiazide (HydroDIURIL). The nurse will assess for which signs and symptoms of digoxin toxicity? Select all that apply. 1. Nausea 2. Yellow vision 3. Irregular pulse 4. Increased urine output 5. Pulse rate of 64 beats per minute

1. Nausea 2. Yellow vision 3. Irregular pulse Signs and symptoms of digoxin toxicity include: bradycardia, headache, dizziness, confusion, nausea, and visual disturbances (blurred vision or yellow vision). In addition, ECG findings may include heart block, atrial tachycardia with block, or ventricular dysrhythmias, all causing an irregular pulse. Increased urine output is an expected effect of the diuretic furosemide; a pulse rate of 64 beats per minute is an acceptable rate when a client is receiving digoxin.

A nurse is caring for a client with chronic obstructive pulmonary disease (COPD). Which clinical finding supports the nurse's suspicion that the client is developing cor pulmonale? 1. Peripheral edema 2. Productive coughing 3. Twitching of the extremities 4. Lethargy progressing to coma

1. Peripheral edema Cor pulmonale is right ventricular failure caused by pulmonary congestion; edema results from increasing venous pressure. A productive cough is symptomatic of the original condition, COPD. Although twitching of the extremities may be caused by alterations in oxygen and hydrogen ion levels and their effects on the central nervous system, it is the sign of peripheral edema that directly indicates increasing venous pressure secondary to cor pulmonale. Although lethargy progressing to coma is caused by alterations in oxygen and hydrogen ion levels and their effects on the central nervous system, the sign of peripheral edema directly indicates increasing venous pressure secondary to cor pulmonale.

An 85-year-old client with a history of congestive heart failure is experiencing dyspnea with a respiratory rate of 32. Crackles are noted bilaterally. The client is in Sim's position, receiving oxygen at 2 L/min via nasal cannula. Which action would the nurse do first? 1. Raise the client to high-Fowler position 2. Obtain the apical pulse and blood pressure 3. Call the primary health care provider immediately 4. Monitor the pulse oximeter to ascertain the oxygen level

1. Raise the client to high-Fowler position Raising the client to high-Fowler position will decrease orthopnea by using gravity to keep fluid in lower extremities, putting less stress on the heart. Obtaining a full set of vital signs would be the next priority after changing the client position. Calling the primary health care provider immediately would not be useful without having a full set of vital signs, which should include the oxygen saturation, which the health care provider would expect the nurse to provide.

A client begins to have difficulty breathing 30 minutes after the insertion of a subclavian central line. What should a nurse do first? 1. Raise the head of the bed. 2. Apply oxygen. 3. Assess breath sounds. 4. Call the primary health care provider requesting a STAT chest x-ray.

1. Raise the head of the bed. The priority is to assist breathing. Raising the head of the bed is the least invasive and first action. Assessing for diminished breath sounds and applying oxygen is important, but should be done after raising the head of the bed. In addition, requesting a chest x-ray, if not already done, is appropriate, but the priority is to immediately perform nursing interventions that will promote ventilation.

The client is diagnosed with peripheral arterial disease (PAD) and the nurse is discussing lifestyle modifications. Which of these is the most beneficial lifestyle modification the nurse should teach this client? 1. Stop smoking 2. Take an aspirin once daily 3. Start a walking program 4. Eat a low-fat, low-cholesterol die

1. Stop smoking In clients with PAD, the goal is to promote vasodilation and prevent vasoconstriction. Complete abstinence from smoking or chewing tobacco products is considered the most effective method of preventing vasoconstriction. Smoking cessation should be encouraged. Although aspirin or other antiplatelets drugs are often prescribed for clients with PAD, it is not the primary means of risk reduction. Although a sedentary lifestyle is a contributing factor, it is not the primary risk factor for PAD. Although a high-fat, high-cholesterol diet is a contributing factor, it is not the primary risk factor for PAD.

A client admitted for uncontrolled hypertension and chest pain was started on a daily diuretic two days ago upon admission, with prescriptions for a daily basic metabolic panel. The client's potassium level this morning is 2.7 mEq/L. Which action should the nurse take next? 1 Notify the health care provider that the potassium level is above normal 2 Notify the health care provider that the potassium level is below normal 3 No action is required because the potassium level is within normal limits 4 Hold the client's morning diuretic dose

2 Notify the health care provider that the potassium level is below normal The health care provider should be notified immediately because the client's potassium is below normal. The normal potassium level range is 3.5 mEq/L--5.0 mEq/L. Clients on diuretic therapy require close monitoring of their electrolytes.

A client arrives at the emergency room complaining of chest pain and dizziness. The client has a history of angina. The health care provider prescribes an electrocardiogram (ECG) and lab tests. A change in which component of the ECG tracing should the nurse recognize as the client actively having a myocardial infarction (MI)? 1 QRS complex 2 S-T segment 3 P wave 4 R wave

2 S-T segment A displacement of the S-T segment is caused by an active ischemic injury in the myocardium. The QRS complex, the P wave, and the R wave are not associated with a myocardial infarction.

A client with coronary artery disease is scheduled for a cardiac catheterization. The nurse's preoperative teaching is considered effective if the client can describe: 1 What will occur if there is an emergency. 2 What will be experienced during the procedure. 3 The risks associated with this invasive procedure. 4 The importance of immediate postoperative exercises.

2 What will be experienced during the procedure. Knowing what to expect reduces fear of the unknown. Knowing what will occur in an emergency may increase fear associated with the experience. Discussing the risks of the procedure is the health care provider's responsibility. The nurse does not give the risks associated with this invasive procedure; the nurse determines the client's knowledge regarding the procedure and documents the client's signature on the consent form. Exercise is not immediate; bed rest is prescribed with operative site immobilization for several hours.

A male client with aortic stenosis is scheduled for a valve replacement in 2 days. He tells the nurse, "I told my wife all she needs to know if I don't make it." What response is most therapeutic? 1. "Men your age do very well." 2. "You are worried about dying." 3. "I know you are concerned, but your surgeon is excellent." 4. "I'll get you a sleeping pill tonight because I know you will need it."

2. "You are worried about dying." "You are worried about dying" is a reflective statement that conveys acceptance and encourages further communication. The response "Men your age do very well" is false reassurance that does not lessen anxiety. The response "I know you are concerned, but your surgeon is excellent" is false reassurance and cuts off communication; this statement does not encourage the client to discuss feelings. The reliance on a pill to help the client in this instance evades the problem and cuts off further communication.

Which client is at greatest risk for the development of a venous thrombosis? 1. A 76-year-old female with a 100 pack per year smoking history and hypertension 2. A 68-year-old male on bed rest following a left hip fracture 3. A 42-year-old female with Factor V Leiden mutation on warfarin (Coumadin) 4. A 59-year-old male who is an intravenous drug user with hyperlipidemia

2. A 68-year-old male on bed rest following a left hip fracture Venous thrombosis is the result of inflammation to a vein, hypercoagulability, or venous stasis, or a combination of the three, known as Virchow's triad. Bed rest and hip fracture are two major risk factors for the development of a thrombosis. While the other options present risk factors (cigarette smoking, clotting disorders, and drug abuse), the combination of the two (venous stasis and vessel injury) result in greatest risk for thrombus development.

The nurse is monitoring a client who is receiving peritoneal dialysis. After the dialysate has infused, the client reports severe respiratory difficulty. What immediate action should the nurse implement? 1. Weigh the client 2. Auscultate breath sounds 3. Obtain STAT arterial blood gases 4. Turn the client on their right side

2. Auscultate breath sounds Lung sounds should be auscultated for signs of fluid overload. Weighing the client will not correct the problem. The pulse oximetry reading and other vital signs may be obtained after the solution is drained and diaphragmatic pressure is decreased. It is not necessary to turn the client on their side.

The nurse is caring for a client who is receiving a thiazide diuretic for hypertension. Which food selected by the client indicates to the nurse that dietary teaching about thiazide diuretics was effective? 1. Apples 2. Broccoli 3. Cherries 4. Cauliflower

2. Broccoli Thiazide diuretics are potassium-depleting agents; broccoli provides 267 mg of potassium per 100 grams. Apples provide 80 to 110 mg of potassium per 100 grams of fruit. Cherries provide 191 mg of potassium per 100 g of fruit. Cauliflower provides 206 mg of potassium per 100 g.

What should the nurse include in a teaching plan for a client taking calcium channel blockers such as Nifedipine (Procardia)? Select all that apply. 1. Reduce calcium intake. 2. Change positions slowly. 3. Report peripheral edema. 4. Expect temporary hair loss. 5. Avoid drinking grapefruit juice.

2. Change positions slowly. 3. Report peripheral edema. 5. Avoid drinking grapefruit juice. Changing positions slowly helps reduce orthostatic hypotension. Peripheral edema may occur as a result of heart failure and must be reported. Grapefruit juice affects the metabolism of calcium channel blockers and should be avoided. Reducing calcium intake is unnecessary because calcium levels are not affected. Hair loss does not occur.

A blood transfusion of packed cells has been prescribed for a client with leukemia. The nurse will complete the following steps in what order? 1. Obtain vital signs and history of transfusions 2. Check health care provider's prescription 3. Check client identification before hanging unit of blood 4. Ascertain that intravenous catheter size is 18 or 20 gauge 5. Change main line solution to normal saline

2. Check health care provider's prescription 1. Obtain vital signs and history of transfusions 4. Ascertain that intravenous catheter size is 18 or 20 gauge 5. Change main line solution to normal saline 3. Check client identification before hanging unit of blood Check the health care provider's prescription to notify blood bank what product will be needed. Obtain the client's baseline vital signs and ask if the client has had previous transfusions and if there were any untoward effects. Ascertaining the intravenous catheter size is at least an 18 will prevent hemolysis of red blood cells. The main line solution must be normal saline 0.9 to flush the line and use as a main line if the blood administration must occur because of a reaction. Other solutions can affect blood, causing it to clot. Checking the client identification and verification of blood product is necessary before proceeding.

A child has been diagnosed with hemophilia type A after experiencing excessive bleeding from a minor trauma. The parents ask what are the symptoms of bleeding for which they should be looking in the future. What are the signs for which to look? Select all that apply. 1 Epistaxis 2 Hematuria 3 Hemarthrosis 4 Easy bruising 5 Frequent fevers 6 Fast clotting of injuries 7 Dark colored tarry stools

1 Epistaxis 2 Hematuria 3 Hemarthrosis 4 Easy bruising 7 Dark colored tarry stools Epistaxis, also known as nosebleeds, is a common symptom of a lack of clotting factor. Hematuria (blood in the urine) may be grossly apparent. The child may experience joint pain and deformities from bleeding into joints. Excessive bruising will occur from bleeding into tissue with seemingly minor injuries. Dark colored tarry consistency stools are indicative of gastrointestinal bleeding. Frequent fevers are not associated with hemophilia. Prolonged clotting times occur with this condition.

A client is seen in the clinic with sickle cell anemia. The primary health care provider has prescribed an iron supplement to treat the client's anemia. The primary concern in regards to giving the supplement is: 1 Giving iron with this condition is contraindicated. 2 Give iron with orange juice to improve absorption. 3 Iron given in liquid form should be given using a straw to prevent staining teeth. 4 The client must be warned that iron usually changes the color of feces to black

1 Giving iron with this condition is contraindicated. Giving iron is contraindicated as sickled cells do not incorporate the iron so it will build up in the body, causing pain, rather than being absorbed. Giving iron with orange juice is correct but not to a person with sickle cell anemia. Liquid iron should be administered with a straw to prevent staining teeth, but not with this condition. Feces will turn dark with iron supplements; however, this client should not be receiving iron.

A client is admitted to the hospital with a diagnosis of lower extremity arterial disease (LEAD). What is the most beneficial lifestyle modification the nurse should teach this client? 1 Stop smoking 2 Control blood glucose 3 Start a walking program 4 Eat a low-fat, low-cholesterol diet

1 Stop smoking Smoking is the single most important risk factor for peripheral arterial diseases, and cessation should be encouraged. Although hyperglycemia is a contributing factor, it is not the primary risk factor for LEAD. Although a sedentary lifestyle is a contributing factor, it is not the primary risk factor for LEAD. Although a high-fat, high-cholesterol diet is a contributing factor, it is not the primary risk factor for LEAD.

The nurse is teaching a client who has asthma about using a peak flow meter. Which statement by the client reflects a correct understanding of the use of a peak flow meter? Select all that apply. 1. "Readings in the green zone mean that my asthma is under control." 2. "If I get a reading in the yellow zone, I need to stop what I'm doing and rest, then recheck in an hour." 3. "If I get a reading in the red zone, then I need to take my reliever drug and have my wife take me to the hospital." 4. "I should check the peak flow readings at least twice a day." 5. "I don't need to check my peak flow readings if I take a reliever drug."

1. "Readings in the green zone mean that my asthma is under control." 3. "If I get a reading in the red zone, then I need to take my reliever drug and have my wife take me to the hospital." 4. "I should check the peak flow readings at least twice a day." Peak flow meters are used to measure how well the client's asthma is controlled. Readings in the green zone mean the asthma is under control; however, readings in the red zone indicate a serious respiratory problem and needs to be addressed immediately. The client will need to take a reliever drug and seek emergency help immediately. Peak flow readings need to be measured twice a day. If a reading in the yellow zone occurs, the client should use the reliever drug and then measure the peak expiratory flow (PEF) again in a few minutes to determine whether the drug is working. Improvement in PEF should be seen. Clients need to check the PEF any time a reliever drug is used to determine the drug's effectiveness.

The student nurse demonstrates correct understanding of anemia related to chronic disease with which statement? 1. "Red blood cells appear normal in size and color; however, there is a decreased amount produced." 2. "The red blood cells have an increased life span with a decrease in normal functioning." 3. "Administration of vitamins B12 and folate will help to treat this type of long-term anemia." 4. "This is the mildest form of anemia and is easily corrected through administration of blood products."

1. "Red blood cells appear normal in size and color; however, there is a decreased amount produced." Anemia of chronic disease results in a decrease in the production of red blood cells (RBCs) in response to chronic inflammation; the red blood cells are normal size, shape, and color. There is a decrease in the lifespan of the RBC and the administration of folate or B12 will not correct the anemia as these levels are generally within normal limits. This form of anemia can be very severe and treatment is directed at identification and management of the underlying cause.

A nurse is caring for a client after a left pneumonectomy for cancer. The nurse palpates the client's trachea routinely. What is the rationale for this nursing intervention? 1. A mediastinal shift may have occurred. 2. Nodular lesions may demonstrate metastasis. 3. Tracheal edema may lead to an obstructed airway. 4. The cuff of the endotracheal tube may be overinflated.

1. A mediastinal shift may have occurred. After a pneumonectomy, the mediastinum may shift toward the remaining lung, or the remaining lung may shift toward the empty space, depending on the pressure within the empty space. Either of these shifts will cause the trachea to move from its usual midline position. The trachea is palpated above the suprasternal notch. Metastatic lesions do not appear rapidly. Tracheal edema cannot be assessed through palpation. The cuff of the endotracheal tube cannot be assessed through palpation of the trachea.

The nurse is caring for a client after a right pneumonectomy for cancer. As part of the assessment, the nurse palpates the client's trachea. What is the rationale for this assessment? 1. A mediastinal shift may have occurred. 2. Subcutaneous emphysema may be present. 3. Tracheal edema may lead to an obstructed airway. 4. The cuff of the endotracheal tube may be underinflated

1. A mediastinal shift may have occurred. After a pneumonectomy, the mediastinum may shift toward the remaining lung, or the remaining lung may shift toward the empty space, depending on the pressure within the empty space. Either of these shifts will cause the trachea to move from its usual midline position; this is known as a mediastinal shift. Subcutaneous emphysema is found by palpation over the lungs and chest areas. Tracheal edema cannot be assessed through palpation. The cuff of the endotracheal tube cannot be assessed through palpation of the trachea.

A client presents to the emergency room with coughing and sudden wheezing. The nurse notes the client is progressing quickly into respiratory distress. The nurse identifies that the client is experiencing: 1. Acute asthma attack. 2. Acute bronchitis. 3. Left-sided heart failure. 4. Cor pulmonale.

1. Acute asthma attack. Symptoms for an acute asthma attack often are wheezing, coughing, dyspnea, and chest tightness. Cough, fever, and fatigue are often symptoms exhibited with acute bronchitis. Fatigue, breathlessness, weakness, shortness of breath, and fluid accumulation in the lungs are often signs of left sided heart failure. Tiring easy, shortness of breath with exertion, lower leg edema, chest pain, and heart palpitations often are exhibited with cor pulmonale.

A client who was in an automobile collision is now in hypovolemic shock. Why is it important for the nurse to take the client's vital signs frequently during the compensatory stage of shock? 1. Arteriolar constriction occurs. 2. The cardiac workload decreases. 3. Contractility of the heart decreases. 4. The parasympathetic nervous system is triggered.

1. Arteriolar constriction occurs. The early compensation of shock is cardiovascular and is reflected in changes in pulse, blood pressure, and pulse pressure; blood is shunted to vital organs, particularly the heart and brain. The cardiac workload will increase, not decrease, as the heart attempts to pump more blood to the vital organs. The heart compensates by increasing its contractility, which will increase, not decrease, the cardiac output. The sympathetic, not parasympathetic, nervous system is triggered to produce vasoconstriction.

A nurse is caring for a client with the diagnosis of right ventricular failure. Which condition unrelated to cardiac disease is the major cause of right ventricular failure? 1. Renal disease 2. Hypovolemic shock 3. severe systemic infection 4. Chronic obstructive pulmonary disease (COPD)

4. Chronic obstructive pulmonary disease (COPD) COPD causes destruction of capillary beds around the alveoli, interfering with blood flow to the lungs from the right side of the heart. As the heart continues to strain against this resistance, heart failure eventually results. Renal disease causes stress on the left side of the heart. Hypovolemic shock will not cause stress on the right side of the heart. Severe systemic infection probably will produce greater stress on the left side of the heart.

After teaching a client about a low fat diet, it is most important for the nurse to document: 1. Client's receptiveness to the education. 2. Family members/significant others were educated as well as the client. 3. Client's weight loss goals. 4. Client's ability to plan a low fat meal

4. Client's ability to plan a low fat meal Documenting that client's ability to plan a low fat meal demonstrates the client's ability to apply the education to their lifestyle. Clients can be receptive to education but not understand it. It helps to include family members or significant others in the education. However, it is most effective if the clients themselves take ownership of their health care plan. Not all clients on a low fat diet need to lose weight.

A Foley catheter was placed with an urimeter for an 85-year-old client with a history of congestive heart failure. The output is 45 mL/hour, cloudy, and has sediment. These findings indicate: 1. Inadequate hourly urine output. 2. The furosemide (Lasix) is causing dehydration. 3. All of the indications are within normal findings. 4. Cloudy urine may be indicative of infection

4. Cloudy urine may be indicative of infection Cloudy urine may be indicative of infection, which is also a risk with Foley catheters. A urinalysis should be performed to confirm or rule out a urinary tract infection. The furosemide may cause dehydration but other findings would be assessed, such as skin turgor. Hourly urine output should be at least 30 mL, which is being surpassed. Urine is expected to be clear amber colored; cloudy is not within expected normal appearance.

A client is admitted to the hospital for medical treatment of bronchopneumonia. What test result should the nurse examine to help determine the effectiveness of the client's therapy? 1. Bronchoscopy 2. Pulse oximetry 3. Pulmonary function studies 4. Culture and sensitivity tests of sputum

4. Culture and sensitivity tests of sputum The aim of therapy is to eliminate the causative agent, which is determined from culture and sensitivity tests of sputum. Bronchoscopy shows the appearance of the bronchi but does not indicate the presence or absence of microorganisms. Pulse oximetry is used to assess for hypoxemia; it does not provide data on the condition of the lung tissue itself or the presence or absence of microorganisms. Pulmonary function studies indicate air volume that may be within the expected range despite the presence of bronchopneumonia.

After receiving streptomycin sulfate for 2 weeks as part of the medical regimen for tuberculosis, the client states, "I feel like I am walking like a drunken seaman." The nurse withholds the drug and promptly reports the problem to the health care provider. Which part of the body does the nurse determine is being affected as indicated by the symptom reported by the client? 1. Pyramidal tracts 2. Cerebellar tissue 3. Peripheral motor end-plates 4. Eighth cranial nerve's vestibular branch

4. Eighth cranial nerve's vestibular branch Streptomycin sulfate is ototoxic and may cause damage to auditory and vestibular portions of the eighth cranial nerve. Pyramidal tracts, cerebellar tissue, and peripheral motor end-plates are not affected by streptomycin.

A health care provider prescribes milrinone (Primacor) for a client with a diagnosis of congestive heart failure who was unresponsive to conventional drug therapy. What is most important for the nurse to do first? 1. Administer the loading dose over 10 minutes. 2. Monitor the ECG continuously for dysrhythmias during infusion. 3. Assess the heart rate and blood pressure continuously during infusion. 4. Have the prescription, dosage calculations, and pump settings checked by a second nurse.

4. Have the prescription, dosage calculations, and pump settings checked by a second nurse. Accidental overdose can cause death. Another nurse should verify accuracy of the prescription, dose, and pump settings to prevent harm to the client. Although administering the loading dose over 10 minutes is an appropriate intervention, it is not the first thing the nurse should do. Although monitoring for dysrhythmias is important because they are common with this medication and may be life threatening, it is not the first thing the nurse should do. Although taking the vital signs continuously during the infusion is important because the dose needs be slowed or discontinued if the blood pressure decreases excessively, it is not the first thing the nurse should do.

Rehabilitation of a client with chronic obstructive pulmonary disease (COPD) involves strategies to decrease hospital admissions and to live a more active life. What should the nurse teach the client to do? 1. Initiate activities to eliminate infection. 2. Inhale during movements that require energy. 3. Implement breathing that uses the thoracic muscles. 4. Incorporate humidification into the home environment.

4. Incorporate humidification into the home environment. Humidification of the environment helps to prevent thickened secretions. Liquefied secretions are easier to expectorate. Measures to prevent infection are essential; however, infections are impossible to eliminate. Exhaling requires less energy than inhaling; therefore, movements that use energy should be done during exhalation. The use of abdominal muscles rather than thoracic muscles improves the client's breathing.

A nurse is caring for a client with a pneumothorax who has a chest tube in place with a closed drainage system. Which of these actions by the nurse is correct? 1. Strip the chest tube periodically 2. Administer the prescribed cough suppressant at the scheduled times 3. Empty and measure the drainage in the collection chamber each shift 4. Keep the drainage system lower than the level of the client's chest

4. Keep the drainage system lower than the level of the client's chest The drainage system is kept below the chest to allow gravity to drain the pleural space. The chest tube should not be stripped because this action can cause negative pressure and damage lung tissue. Cough suppressants are not indicated because coughing and deep breathing are encouraged to help reexpand the lung. The closed system is not entered for emptying; when full, the entire device is replaced.

A client with bilateral varicose veins of the lower extremities questions the nurse about the brownish discoloration of the lower legs. The best response by the nurse is, "This is probably the result of: 1. Inadequate arterial blood supply." 2. Delayed healing of tissues after an injury." 3. Increased production of melanin in the area." 4. Leakage of red blood cells through the vascular wall."

4. Leakage of red blood cells through the vascular wall." Increased venous pressure alters the permeability of the veins, allowing extravasation of red blood cells (RBCs); hemolysis of RBCs releases a pigment called hemosiderin, which causes a characteristic brownish discoloration (brawny appearance). The arterial circulation is not affected by the pathology of varicose veins. Although tissue healing may be delayed, the brownish discoloration is the result of the hemolysis of RBCs, not trauma. There is no increase in melanocyte activity in the skin surrounding varicose veins.

A nurse assesses a client for orthostatic hypotension. The results are: Lying - heart rate = 70 beats/minute B/P - 110/70 Sitting - heart rate = 78 beats/minute B/P - 106/66 Standing - heart rate = 85 beats/minute B/P - 100/64 The nurse would expect which primary health care provider prescription? 1. Increase furosemide (Lasix) from 20 mg by mouth (PO) to 40 mg PO daily. 2. Give 1 L of 0.9% normal saline (NS) bolus over 4 hours. 3. Start an IV of D5 ½ NS to run at 150 mL/hr. 4. No prescription change.

4. No prescription change. The assessment findings do not indicate postural hypotension. There is no indication from the data that a prescription change is needed for this client. Increasing the furosemide or giving IV fluid to this client could result in a fluid imbalance in this client.

Before discharge, the nurse is planning to teach the client with emphysema pursed-lip breathing. The nurse should instruct the client that the purpose of pursed-lip breathing is to: 1. Decrease chest pain 2. conserve energy 3. Increase oxygen saturation 4. Promote elimination of CO2

4. Promote elimination of CO2 Pursed-lip breathing increases positive pressure within the alveoli and makes it easier for clients to expel air from the lungs. This in turn promotes elimination of CO2. It also helps clients to slow their breathing pattern and depth with respirations. It does not decrease chest pain, conserve energy, or increase oxygen saturation.

The health care provider has prescribed intravenous (IV) normal saline at 200 mL/hr and furosemide (Lasix) 120 mg orally stat for an 85-year-old client with a history of congestive heart failure. The best response is: 1. Place the normal saline on an infusion pump to control the amount and give the furosemide. 2. Ask the health care provider why so much intravenous fluid is to be given to an elderly client, and give the furosemide as prescribed. 3. Decline to give the intravenous fluid saying it could cause circulatory overload, and give the furosemide as prescribed. 4. Question the choice of solution, the amount to be given, and the dose of furosemide that has been prescribed.

4. Question the choice of solution, the amount to be given, and the dose of furosemide that has been prescribed. The choice of normal saline at 200 mL/hr should be questioned for using saline, which is inclined to retain fluid, and the amount, which would be too much for most elderly persons' cardiac status to tolerate. This client is in congestive heart failure as evidenced by crackles and dyspnea and pulmonary edema is occurring. The standard initial dose of furosemide is 20-80mg. Giving a higher dose needs to be questioned and the nurse can refuse to follow prescriptions that are outside of standards. Using an infusion pump to infuse solution assures the prescribed amount is infused but does not address that this is too much. Giving a medication without understanding normal range and information violates standards. In addition, elderly clients do not metabolize medication as efficiently as younger clients.

The client is receiving multiple blood transfusions after having extensive abdominal surgery. If the client develops fever, chills, and lower back pain, and seems very nervous, what will be the nurse's first action? 1. Notify the blood bank. 2. Notify the health care provider. 3. Reduce the rate of the blood transfusion. 4. Stop the blood and infuse normal saline.

4. Stop the blood and infuse normal saline. Fever, chills, and lower back pain, along with apprehension, headache, tachycardia, and hypotension, indicate an acute hemolytic reaction, which is potentially life threatening. Discontinuing the transfusion immediately limits kidney damage. The vein is kept open by running the primary bag of normal saline. Although the blood bank will be notified if a reaction occurs, it can be done later. Notifying the health care provider can done by another nurse. The client's safety must be addressed first. Slowing the transfusion rate is unsafe because the reaction will continue.

A nurse assesses a client's intravenous site. What clinical finding, unique to infiltration, leads the nurse to conclude that the intravenous (IV) site has infiltrated, rather than become inflamed? 1. Pain 2. Coolness 3. Localized swelling 4. Cessation in flow of solution

2. Coolness When an IV infiltrates, the IV solution entering the interstitial space is at room temperature (approximately 75° F), whereas body temperature is approximately 98.6° F; therefore, the client's skin will feel cool to the touch at the site of an IV infiltration. The site of an inflammation will feel warm to the touch because of vasodilation and hyperemia. Pain may occur with both an inflammation and an infiltration. The pain of an inflammation is related to the pressure of edema on nerve endings. The pain of an infiltration is related to the IV solution in the interstitial compartment pressing on nerve endings. An increase in interstitial fluid occurs with both an inflammation and an infiltration. With an inflammation there is increased vascular permeability at the site; fluid, proteins, and leukocytes then move from the intravascular compartment into the interstitial compartment. With an infiltration the IV solution enters the interstitial compartment rather than the intravascular compartment. A cessation in flow of solution occurs with both an inflammation and an infiltration. An inflammation in the vein at the insertion site may close the lumen of the vessel interfering with the flow of solution. An infiltration will cause excess fluid in the interstitial compartment to the extent that it will not accommodate more solution, interfering with the flow of the solution.

A nurse assesses a client's intravenous site. What clinical finding leads the nurse to conclude that the intravenous (IV) site has been infiltrated? Select all that apply. 1. Redness along the vein 2. Coolness of skin near the insertion site 3. Swelling around the insertion site 4. Cessation in flow of solution 5. Vein feels hard and cordlike

2. Coolness of skin near the insertion site 3. Swelling around the insertion site 4. Cessation in flow of solution When an IV infiltrates, the IV solution entering the interstitial space is at room temperature (approximately 75° F), whereas body temperature is approximately 98.6° F; therefore, the client's skin will feel cool to the touch at the site of an IV infiltration. In addition, the fluid in the interstitial space causes swelling around the insertion site, and the solution stops flowing. Redness along the vein, with the vein feeling hard and cordlike, is present with phlebitis.

A 28-year-old male client is undergoing tests to confirm the diagnosis of Hodgkin lymphoma. The client and his wife are worried that he may have cancer. The wife states, "Don't you think it is unlikely for someone like my husband to have cancer?" The nurse's response is based on the information that Hodgkin lymphoma is: 1. More likely to affect women than men. 2. Diagnosed during adolescence and young adulthood. 3. Typically a disease of older rather than younger adults. 4. Usually occurs frequently among populations of Asian heritage.

2. Diagnosed during adolescence and young adulthood. Hodgkin lymphoma occurs most often during the ages of 15 to 35 years of age and between 50 to 60 years of age. Hodgkin lymphoma affects younger men and women equally and affects more men than women between the ages of 50 and 60 years. The incidence of Hodgkin lymphoma is not limited to people in older age groups. The prevalence of Hodgkin lymphoma is increased in teenagers and young adults (15 to 35 years of age). Asian populations are less likely to develop Hodgkin lymphoma than other populations.

A client is admitted to the hospital with a diagnosis of an exacerbation of asthma. What should the nurse plan to do to best help this client? 1. Determine the client's emotional state. 2. Give prescribed drugs to promote bronchiolar dilation 3. provide education about the impact of a family history 4. encourage the client to use an incentive spirometer routinely

2. Give prescribed drugs to promote bronchiolar dilation sthma involves spasms of the bronchi and bronchioles, as well as increased production of mucus; this decreases the size of the lumina, interfering with inhalation and exhalation. Bronchiolar dilation will reduce airway resistance and improve the client's breathing. Although identifying and addressing a client's emotional state is important, maintaining airway and breathing are the priority. In addition, emotional stress is only one of many precipitating factors, such as allergens, temperature changes, odors, and chemicals. Although recent studies indicate a genetic correlation along with other factors that may predispose a person to develop asthma, exploring this issue is not the priority. Use of an incentive spirometer is not helpful because of mucosal edema, bronchoconstriction, and secretions, all of which cause airway obstruction.

A client has arrived on the nursing unit scheduled for a cholecystectomy, and there is one registered nurse, a licensed practical nurse, and a certified nursing assistant. Institution policy is that all admissions must have the initial assessment completed by a registered nurse. The registered nurse currently is caring for a client with a transfusion reaction. Which is the best decision? 1. Tell the practical nurse to admit the patient and complete documentation. 2. Have the practical nurse perform the initial assessment and the registered nurse verify all findings after the transfusion client is stable. 3. Instruct the certified nursing assistant to obtain vital signs every 15 minutes as this is within the scope of practice. 4. Tell the admission clerk to put the client in the designated room and explain that the nurses are busy with an emergency and will get to the client as soon as possible.

2. Have the practical nurse perform the initial assessment and the registered nurse verify all findings after the transfusion client is stable. The practical nurse is able to perform the initial assessment but the registered nurse must verify and sign for all findings. The transfusion client is a priority until stable and warrants care by the registered nurse. Institution policy dictating initial assessment by the registered nurse will be met by this plan. Telling the practical nurse to do the assessment and completing documentation does not include verification by the registered nurse, which is in the institution policy. The certified nursing assistant is capable of taking vital signs but does not have the assessment skills necessary to care for an acute client. Instructing the admission personnel to place a client in a room and stating that the nurses are too busy would not inspire confidence.

The client with emphysema complains of increased shortness of breath and becomes anxious. The health care provider prescribes oxygen at 1 L/minute via nasal cannula. The nurse understands that this prescription is appropriate because: 1. High concentrations of oxygen cause alveoli to rupture. 2. High concentrations of oxygen eliminate the respiratory drive. 3. The client does not need any more than 1 L/minute. 4. The oxygen at 1 L/minute should be enough to diminish the anxiety.

2. High concentrations of oxygen eliminate the respiratory drive. Clients with emphysema are used to low levels of oxygen and high levels of carbon dioxide. Oxygen is the stimulus for breathing for these clients instead of the natural breathing stimulus. Too much oxygen will knock out the stimulus to breathe. High concentrations of oxygen will not cause a rupture. The client actually could need more oxygen; however, if a higher concentration is given, it will knock out the respiratory drive. The oxygen is being given because of the shortness of breath.

A child with a congenital heart defect has a cardiac catheterization. What is an essential element of nursing care after this procedure? 1. Encouraging early ambulation 2. Monitoring the extremity distal to the insertion site 3. Restricting fluids until blood pressure and heart rate have stabilized 4. Comparing blood pressure in the affected and unaffected extremities

2. Monitoring the extremity distal to the insertion site Monitoring the extremity distal to the insertion site for changes in temperature and color should indicate the presence or absence of a clot; comparing pedal pulses of both extremities may reveal clot formation that disrupts circulation. The child is kept in bed at least 6 hours after the procedure. Fluids may be given as soon as tolerated. Pulses, not blood pressure, must be checked for quality and symmetry.

An 80-year-old client with a history of coronary artery disease is admitted to the hospital for observation after a fall. During the night the client has an episode of paroxysmal nocturnal dyspnea. In what position should the nurse place the client to best decrease preload? 1. Contour 2. Orthopneic 3. Recumbent 4. Trendelenburg

2. Orthopneic The client's paroxysmal dyspnea was probably caused by sleeping in bed with the legs at the level of the heart; this position increases venous return from dependent body areas, increasing the intravascular volume. Sitting up and leaning forward while keeping the legs dependent slows venous return as well as increases thoracic capacity. Although the contour position elevates the client's head, it does not place the legs in a dependent enough position to substantially decrease venous return. The recumbent position is contraindicated. Venous return increases when the lower extremities are at the level of the heart. Also, the pressure of the abdominal organs against the diaphragm decreases thoracic capacity. The Trendelenburg position is contraindicated. Venous return increases when the lower extremities are higher than the level of the heart. Also, the pressure of the abdominal organs against the diaphragm decreases thoracic capacity.

The nurse who is teaching the client with chronic obstructive pulmonary disease (COPD) to use pursed-lip breathing (PLB) is aware that this is beneficial for the client through which mechanism? 1. Increased respiratory rate to improve arterial oxygenation 2. Prolonged exhalation to decrease air trapping 3. shortened inhalation to reduce bronchial swelling 4. Use of the diaphragm to increase the amount of inspired air

2. Prolonged exhalation to decrease air trapping Pursed-lip breathing works to decrease dyspnea and the respiratory rate through prolonging exhalation and prevention of alveolar collapse. PLB does not increase the length of inhalation and does not increase the respiratory rate. Use of the diaphragm occurs with diaphragmatic, or abdominal, breathing.

A nurse is caring for a client with a history of chronic obstructive pulmonary disease (COPD) who develops a pneumothorax and has a chest tube inserted. What is the primary purpose of the chest tube? 1. Lessens the client's chest discomfort 2. Restores negative pressure in the pleural space 3. Drains accumulated fluid from the pleural cavity 4. Prevents subcutaneous emphysema in the chest wall

2. Restores negative pressure in the pleural space Negative pressure is exerted by gravity drainage or by suction through the closed system. Though the discomfort may be lessened as a result of the insertion of the chest tube, this is not the primary purpose. There is an accumulation of air, not fluid, when a pneumothorax occurs in a client with COPD. Subcutaneous emphysema in the chest wall is associated most commonly with clients receiving air under pressure, such as that received from a ventilator.

A nurse administers leucovorin calcium to a client before the prescribed methotrexate (Trexall). The client asks the reason for this. What effect of leucovorin calcium should the nurse consider when formulating a response? 1. Potentiates metabolite required for destruction of cancer cells 2. Supplies levels of folic acid required by blood-forming organs 3. Acts synergistically with antineoplastic drugs to destroy cancer cells 4. Increases production of phagocytes to help remove debris from destroyed cancer cells

2. Supplies levels of folic acid required by blood-forming organs Methotrexate (Trexall) is a folic acid antagonist that can depress the bone marrow; this serious toxic effect sometimes is prevented by administration of folic acid. Some health care providers advocate its administration after a course of methotrexate therapy to avoid interfering with methotrexate activity. Folic acid is a metabolite and does not destroy cancer cells. Leucovorin calcium does not increase the production of phagocytes.

A nurse provides instruction when the beta blocker atenolol (Tenormin) is prescribed for a client with moderate hypertension. What action identified by the client indicates to the nurse that the client needs further teaching? 1. Move slowly when changing positions. 2. Take the medication before going to bed. 3. Expect to feel drowsy when taking this drug. 4. Count the pulse before taking the medication.

2. Take the medication before going to bed. Beta blockers (BBs) should not be taken at night because the blood pressure usually decreases when sleeping. This medication blocks beta-adrenergic receptors in the heart, which ultimately lowers the blood pressure. Therefore, the drug should be taken early in the morning to maximize its therapeutic effect. Orthostatic hypotension is a side effect of BBs, and the client should change positions slowly to prevent dizziness and falls. Drowsiness is a side effect of BBs, and the client should be taught precautions to prevent injury. The pulse rate should be taken before administration because ventricular dysrhythmias and heart block may occur with BBs.

After resection of a lower lobe of the lung, a client has excessive respiratory secretions. Which independent nursing action should the nurse implement? 1. Postural drainage 2. Turning and positioning 3. Administration of an expectorant 4. Percussion and vibration techniques

2. Turning and positioning Turning and positioning does not require a health care provider's prescription and is an independent action. Postural drainage, administration of an expectorant, and percussion and vibration techniques are dependent nursing functions that require a health care provider's prescription.

A client develops subcutaneous emphysema after a chest injury with suspected pneumothorax. What assessment by the nurse is the best method for detecting this complication? 1. Percussing the neck and chest 2. palpating the neck or face 3. auscultating for abnormal breath sounds 4. observing for asymmetry of chest movement

2. palpating the neck or face Subcutaneous emphysema refers to the presence of air in the tissue that surrounds an opening in the normally closed respiratory tract; the tissue appears puffy, and a crackling sensation is detected when trapped air is compressed between the nurse's palpating fingertips and the client's tissue. Percussion is not an appropriate method for assessment; breath sounds are not affected. Asymmetry of chest movements may occur because of the pneumothorax but are not indicative of subcutaneous emphysema.

A client who is suspected of having had a silent myocardial infarction has an electrocardiogram (ECG) prescribed by the health care provider. While the nurse prepares the client for this procedure, the client asks, "Why was this test prescribed?" The best reply by the nurse is, "This test will 1. Detect your heart sounds 2. reflect any heart damage 3. help us change your heart's rhythm 4. Tell us how much stress your heart can tolerate

2. reflect any heart damage Changes in an ECG will reflect the area of the heart that is damaged because of hypoxia. A stethoscope is used to detect heart sounds. Medical interventions, such as cardioversion or cardiac medications, not an ECG, can alter heart rhythm. An ECG will reflect heart rhythm, not change it. Identifying how much stress a heart can tolerate is accomplished through a stress test; this uses an ECG in conjunction with physical exercise.

During administration of a whole blood transfusion, the client begins to complain of shortness of breath. The nurse notes the presence of jugular venous distension, bibasilar crackles, and tachycardia. Prioritize the following nursing actions. 1.Reduce the flow rate of the transfusion 2.Elevate the head of the bed to 45 degrees 3.Apply oxygen via nasal cannula 4.Administer furosemide (Lasix) per provider prescription 5.Document findings in the client record

2.Elevate the head of the bed to 45 degrees 3.Apply oxygen via nasal cannula 1.Reduce the flow rate of the transfusion 4.Administer furosemide (Lasix) per provider prescription 5.Document findings in the client record These symptoms represent circulatory overload. First, the nurse's priority is to facilitate gas exchange by elevating the head of the bed, then applying oxygen. Next, the transfusion rate should be slowed to reduce further circulatory overload and client compromise, followed by the administration of a diuretic to reduce circulating volume. Lastly, the findings and interventions should be documented accordingly.

A client is admitted to the hospital for an emergency cardiac catheterization. What is the most common adaptation that the client is most likely to complain about after this procedure? 1 Fear of dying 2 Skipped heartbeats 3 Pain at the insertion site 4 Anxiety in response to intensive monitoring

3 Pain at the insertion site Pain at the arterial puncture site is attributable to entry and cannulation of the artery and is a common complaint after a cardiac catheterization. Fear of dying might occur during the precatheterization period. Although skipped heartbeats may occur during the procedure because of trauma to the conduction system, usually it does not continue after the procedure. Although some clients may be anxious, many feel safe when receiving ongoing monitoring.

A health care provider prescribes a diuretic for a client with hypertension. What should the nurse include in the teaching when explaining how diuretics reduce blood pressure? 1 Facilitates vasodilation 2 Promotes smooth muscle relaxation 3 Reduces the circulating blood volume 4 Blocks the sympathetic nervous system

3 Reduces the circulating blood volume Diuretics block sodium reabsorption and promote fluid loss, decreasing blood volume and reducing arterial pressure. Direct relaxation of arteriolar smooth muscle is accomplished by vasodilators, not diuretics. Vasodilators, not diuretics, act on vascular smooth muscle. Drugs that act on the nervous system, not diuretics, inhibit sympathetic vasoconstriction.

The nurse caring for a client admitted for chest pain and a myocardial infarction (MI) is preparing to apply nitroglycerin ointment. Before applying, the nurse should: 1 Assess the client's pulse rate. 2 Prepare the site with an alcohol swab. 3 Remove ointment previously applied. 4 Expect the client to be relieved of pain within 20 minutes.

3 Remove ointment previously applied. Before applying the nitroglycerin ointment, the nurse should remove the previous ointment. If the previously applied ointment is not removed, the client could receive too much medication. The nurse should assess blood pressure reading, not pulse rate. There is no need to clean the site with alcohol before administration. Nitroglycerin ointment is not used to treat acute pain.

A client is to receive conscious sedation during a cardiac catheterization. Which route of administration should the nurse explain will be used to deliver the conscious sedation? 1 Via a face mask 2 Into the epidural space 3 Through an intravenous catheter 4 Around the nerves innervating the chest wall

3 Through an intravenous catheter Conscious sedation is administered by direct intravenous (IV) injection or IV push to dull or reduce the intensity of pain or awareness of pain during a procedure without loss of defensive reflexes. General anesthesia usually is administered via inhalation of the vapor of a volatile liquid or an anesthetic gas via a mask or endotracheal tube; as a result, the client is unconscious, unaware, and anesthetized. An epidural block, a type of regional anesthesia, involves the injection of a local anesthetic into the epidural (extradural) space; it works by binding to nerve roots as they enter and exit the spinal cord. A nerve block, a type of regional anesthesia, is achieved by injection of the anesthetic agent into or around the nerves supplying the area; it interrupts sensory, motor, and sympathetic transmission.

The nurse is preparing to discharge a client who presented to the emergency room for an acute asthma attack. The nurse notes that upon discharge the health care provider has prescribed theophylline (Theo-Dur) 300 mg orally to be taken daily at 9:00 AM. The nurse should teach the client to take the medication: 1. With a meal 2. Only at bedtime 3. At a specific time prescribed 4. Until symptoms are gone.

3. At a specific time prescribed For theophylline to be effective, therapeutic serum levels must be maintained by taking the medication at the prescribed time. If the medication is not taken at the prescribed time, the level may drop below the therapeutic range. The medication will not be effective if it drops below the therapeutic range. Theophylline should be given after a meal and with a full glass of water to decrease gastric irritability. It should not be taken at night as it can cause central nervous system stimulation resulting in insomnia, restlessness, irritability, etc. Theophylline is used for long term medication therapy.

A client who is to be admitted for minor surgery has a chest radiograph as part of the presurgical physical. The nurse is notified that the radiograph reveals that the client has pulmonary tuberculosis. What evidence of tuberculosis is provided by the radiograph? 1. Sensitized T cells 2. Presence of acid-fast bacilli 3. Cavities caused by caseation 4. Microscopic primary infection

3. Cavities caused by caseation Cavities are evident on radiograph. Necrotic lung tissue may liquefy, leaving a cavity (cavitation), or granulose tissue can surround the lesion, become fibrous, and form a collagenous scar around the tubercle (Ghon tubercle). Sensitized T cells are determined by a positive reaction to a tuberculin skin test, not on radiograph; a skin test only determines the presence of antibodies; it does not confirm active disease. Presence of acid-fast bacilli may be determined by a sputum culture, not by radiograph. Microscopic primary infection may be so small it does not appear on a radiograph.

The nurse observes a client collapse while walking down the hallway. What will the nurse do first? 1. Do a blind finger sweep 2. Begin chest compressions 3. Check for a carotid pulse 4. Perform the abdominal thrust maneuver

3. Check for a carotid pulse According to the 2010 American Heart Association guidelines, assessing for a carotid pulse is the first step in CPR. A blind finger sweep is not performed. Chest compressions are done only after it is determined that the carotid pulse is absent. The abdominal thrust (Heimlich) maneuver is used to relieve airway obstruction and is not appropriate in this instance.

A client, receiving a potassium infusion via a peripheral intravenous (IV) site, complains of burning sensation above the IV site. What should the nurse do first? 1. Apply warm compresses to the affected extremity. 2. Slow the IV infusion until the burning sensation is gone. 3. Check the IV access for a blood return. 4. Call the primary health care provider and request the IV infusion be changed to an oral supplement.

3. Check the IV access for a blood return. Because potassium infusions can be caustic to the vein, a nurse should check for continued blood return. That finding determines the nurse's next intervention(s). If blood return is present then it is appropriate to apply warm compresses and slow the IV infusion to decrease the burning sensation. If there is not a blood return, the infusion needs to be stopped via that IV site. If the potassium infusion cannot be administered, the primary health care provider must be notified so that other means of potassium replacement can be instituted.

A blood transfusion of packed cells has been prescribed for a client. The nursing unit is extremely busy. In delegating the task of blood administration: 1. Assign a licensed practical nurse (LPN) and a nursing assistant to verify the blood is correct and have the LPN monitor the client 15 minutes after hanging the product. 2. Have two registered nurses ascertain that the client identification and blood product are correct with no discrepancies, hang the blood, and check in 15 minutes. 3. Have all identification verified by registered nurses, then the registered nurse can hang the product and monitor client, staying with client during the initial 15 minutes. 4. Have the product and name band verified by a registered nurse, hang, and monitor every hour until infused within a 10-hour period or discontinued.

3. Have all identification verified by registered nurses, then the registered nurse can hang the product and monitor client, staying with client during the initial 15 minutes. The blood product must be checked by two registered nurses, and client identification verified. A registered nurse hangs the product and should stay in the room for at least the initial 15 minutes, at which time a reaction is most likely and if it occurs the transfusion can be stopped. Assigning an LPN to administer blood is not within the scope of practice in some states, and the product should be checked by two registered nurses. Checking in 15 minutes is too long if the client has a hemolytic reaction so the nurse should stay with the client and obtain vital signs. Monitoring every hour, especially during the first hour, is too long. Packed cells usually are infused in 2 hours and cannot be hung longer than 4 hours.

Valsartan (Diovan), an angiotensin II receptor antagonist, is prescribed for a client. For which possible side effects should the nurse monitor the client? Select all that apply 1. Constipation 2. Hypokalemia 3. Irregular pulse rate 4. Change in visual acuity 5. Orthostatic hypotension

3. Irregular pulse rate 5. Orthostatic hypotension Dysrhythmias, including second-degree heart block, are cardiovascular side effects of valsartan. It also may precipitate angina pectoris, myocardial infarction, and brain attack (cerebrovascular accident, CVA). Angiotensin II receptor antagonists, such as valsartan, block vasoconstrictor and aldosterone-producing effects of angiotensin II at receptor sites, including vascular smooth muscle, thus reducing the blood pressure; dizziness, orthostatic hypotension, and excessive hypotension may occur. Diarrhea, not constipation, may occur with valsartan. Hyperkalemia, not hypokalemia, may occur with valsartan. Valsartan does not cause altered visual acuity.

A nurse is teaching a client with hypertension about a sodium-restricted diet. What information should the nurse emphasize? 1. Using salt-free natural seasonings can taste the same as salt. 2. Desiring the taste for salt is inherent but can be overcome with practice. 3. Liking the taste of table salt is learned but it is not a biological necessity. 4. Substituting table salt with potassium chloride can be done freely.

3. Liking the taste of table salt is learned but it is not a biological necessity. The taste for salt is learned from habitual use and can be unlearned or reduced with health improvement motivation and creative salt-free food preparation. Substitutes do not taste the same as salt. The taste for salt is learned. Using salt substitutes containing potassium chloride may be unsafe; excessive use can produce abnormally high serum potassium levels.

A client with pneumonia now requires use of a nonrebreathing mask to maintain adequate oxygen saturation levels. How does the nurse interpret this information? 1. The client's pneumonia is continually improving 2. Oxygen concentrations up to 44% can be obtained 3. Mechanical ventilation may be required next 4. Nasal cannula may be used while the client is eating

3. Mechanical ventilation may be required next A nonrebreathing mask is used when the client requires higher oxygen concentrations and the condition is worsening. If the nonrebreathing mask does not improve oxygen saturation, the next step to improving gas exchange and oxygenation is intubation and mechanical ventilation. Oxygen concentrations up to 90% can be achieved. Nasal cannula would not be advised as the client requires more oxygen than can be delivered through this method.

A health care provider prescribes losartan (Cozaar) for a client. Which is the most important nursing action? 1. Assess the client for hypokalemia. 2. Ensure that the medication is ingested with food. 3. Monitor the client's blood pressure during therapy. 4. Teach that a missed dose can be doubled at the next scheduled time.

3. Monitor the client's blood pressure during therapy. Losartan is an antihypertensive. It blocks vasoconstrictor and aldosterone-producing effects of angiotensin II at receptor sites. A lowering of the client's blood pressure reflects a therapeutic response and should be monitored frequently. The client may be at risk for hyperkalemia, not hypokalemia. Losartan may be taken without regard to meals. Doubling a dose is unsafe. A missed dose can be taken as long as it is not close to the next scheduled dose.

A client that is postoperative hip replacement is receiving morphine by patient-controlled analgesia and has a respiratory rate of 6 breaths/min. What intervention should the nurse anticipate? 1. Nasotracheal suction 2. Mechanical ventilation 3. Naloxone administration 4. Cardiopulmonary resuscitation

3. Naloxone administration Naloxone is an opioid (narcotic) antagonist and will reverse respiratory depression caused by opioids. Nasotracheal suction, mechanical ventilation, and cardiopulmonary resuscitation are not needed; naloxone will correct the respiratory depression.

A client who has a hemoglobin of 6 gm/dL is refusing blood because of religious reasons. What is the most appropriate action by the nurse? 1. Call the chaplain to convince the client to receive the blood transfusion. 2. Discuss the case with coworkers. 3. Notify the primary health care provider of the client's refusal of blood products. 4. Explain to the client that they will die without the blood transfusion.

3. Notify the primary health care provider of the client's refusal of blood products. The nurse serves as an advocate for the client to uphold their wishes. Synthetic blood products are available but must be prescribed by the primary health care provider. Therefore, the primary health care provider needs to be notified of the client's refusal for blood so alternatives can be considered. . The chaplain's role is to offer support, not to convince the client to go against their beliefs. It is a HIPAA violation to discuss the case with coworkers unless they are involved in the care of the client. The nurse should not use threats or fear to coerce the client.

After teaching a family member how to administer subcutaneous enoxaparin sodium (Lovenox), how should a nurse evaluate the effectiveness of the training? 1. Return demonstration on a manikin. 2. Verbalization of the side effects of the medication. 3. Observing the family member administering enoxaparin sodium to the client. 4. Correctly verbalizing all necessary steps in enoxaparin sodium administration.

3. Observing the family member administering enoxaparin sodium to the client. The best way to evaluate the effectiveness of the teaching is to observe the family member administering the medication to the client. The family member may be able to perform a subcutaneous injection on a manikin, but fear hurting their family member. Knowing the side effects of enoxaparin sodium is important, but it does not provide any information as to their ability to administer the medication. The family member may be able to verbalize all the steps, but fear puncturing the skin with the needle.

A health care provider prescribes daily sputum specimens to be collected from a client. When is the most appropriate time for the nurse to collect these specimens? 1. After activity 2. Before meals 3. On awakening 4. Before a respiratory treatment

3. On awakening During sleep, mucous secretions in the respiratory tract move slowly toward the throat. On awakening, increased ciliary motion raises these secretions more vigorously, thus facilitating expectoration and the collection of sputum specimens. Although activity mobilizes secretions, no secretions may be present at the time of activity; sputum is most plentiful upon arising. The sputum may leave an unpleasant taste in the mouth, which may interfere with appetite. Sputum more likely would be collected after, not before, a respiratory treatment, because this mobilizes secretions.

How should the nurse position a client in the immediate postoperative period after a right pneumonectomy? 1. In high-Fowler 2. Flat in bed with the knees flexed 3. On the right side with the head elevated 4. With the head of the bed elevated in left Sims

3. On the right side with the head elevated To maintain expansion of the remaining lung after a pneumonectomy, the client should be positioned on the operative side or the back. A high-Fowler position may cause the client to slip down in bed, diminishing thoracic excursion. Keeping the client flat will decrease lung expansion; flexing the knees may cause peripheral circulatory complications. The client should not be placed on the unaffected side; this will impede lung expansion.

An 85-year-old client with a history of congestive heart failure on daily weights has a 2-pound weight gain and pitting edema in lower extremities bilaterally, indicative of fluid retention. The most appropriate response is: 1. Check the record to ascertain the code status 2. Notify the primary health care provider of the change 3. Perform a head-to-toe assessment, including vital signs 4. Continue to monitor daily weights and edema and document findings

3. Perform a head-to-toe assessment, including vital signs Performing a head-to-toe assessment, including vital signs, would indicate symptoms, such as jugular distention with right sided heart failure, or pulmonary issues associated with left-sided heart failure. Checking the record for code status is not a priority and should have been established and known on an elderly client. Notifying the health care provider of the weight change and edema without full vital signs and assessment would not be helpful as this information will be needed. Continuing to monitor daily weights without an assessment may miss worsening symptoms.

A client who has experienced a fracture of the femur is experiencing respiratory difficulties, and the nurse suspects a pulmonary embolus. Which of these assessment findings is specific to a fat embolism? 1. Chest pain 2. Dyspnea 3. Petechiae 4. Decreased SaO2

3. Petechiae Petechiae on the chest and shoulders suggest fat emboli after fractures. The petechial rash occurs from occlusion of small dermal capillaries, leading to extravasation of red blood cells. Both a fat embolism and a blood clot embolism in the lungs may cause symptoms, such as altered mental status, chest pain, dyspnea, decreased SaO2, and increased respirations and pulse.

During the assessment of a client who was admitted to the hospital because of a productive cough, fever, and chills, the nurse percusses an area of dullness over the right posterior lower lobe of the lung. The nurse determines that the client's signs and symptoms may be indicative of: 1. Pleurisy. 2. Bronchitis. 3. Pneumonia. 4. Emphysema.

3. Pneumonia. The data presented indicate an infectious process within the lung. The classic clinical findings associated with pneumonia are a productive cough (sputum is purulent, blood-tinged, or rust-colored), fever, chills, pleuritic chest discomfort, and dyspnea. Percussion is dulled over areas of consolidation. The cardinal clinical findings associated with pleurisy are pain in the lower lobe at the height of inspiration and a pleural friction rub. Although fever and chills can occur later in bronchitis, the cardinal clinical findings associated with bronchitis are irritating cough, chest pain, and shortness of breath. The cardinal clinical findings associated with emphysema are barrel chest, resonance on percussion, and thick, tenacious sputum.

When providing discharge teaching for a young female client who had a pneumothorax, it is important that the nurse include the signs and symptoms of a recurring pneumothorax. What is the most important symptom that the nurse should teach the client to report to the healthcare provider? 1. Substernal chest pain 2. Episodes of palpation 3. Severe shortness of breath 4. Dizziness when standing up

3. Severe shortness of breath Severe shortness of breath may indicate a recurrence of the pneumothorax because one lung is unable to meet the oxygen demands of the body. A pneumothorax causes sharp chest pain on the involved side, not substernally. Usually palpitations reflect a cardiac, not a respiratory, problem. Dizziness when standing up is not specific to a pneumothorax; this is orthostatic hypotension, which may be related to a variety of medical problems.

The nurse is making rounds on a client who has developed severe bone marrow depression after receiving chemotherapy for cancer. Which of these actions by the nurse is appropriate? Select all that apply. 1. Monitor for signs of alopecia. 2. Encourage an increase in fluids. 3. Wash hands before entering the client's room. 4. Advise use of a soft toothbrush for oral hygiene. 5. Report an elevation in temperature immediately. 6. Encourage the client to eat raw, fresh fruits and vegetables.

3. Wash hands before entering the client's room. 4. Advise use of a soft toothbrush for oral hygiene. 5. Report an elevation in temperature immediately. It is essential to prevent infection in a client with severe bone marrow depression; thorough hand-washing before touching the client or client's belongings is important. Thrombocytopenia occurs with most chemotherapy treatment programs; using a soft toothbrush helps prevent bleeding gums. Any temperature elevation in a client with neutropenia must be reported to the health care provider immediately as it may be a sign of infection. Although alopecia does occur with chemotherapy, it is not related to bone marrow suppression. Increasing fluids will neither reverse bone marrow suppression nor stimulate hematopoiesis. This is not related to bone marrow suppression. Clients who have severe bone marrow depression must avoid eating raw fruits and vegetables, and undercooked meat, eggs, and fish to avoid possible exposure to microbes.

The health care provider prescribes an intravenous medication for a client who has been admitted for a chronic obstructive pulmonary disease (COPD) exacerbation. When preparing to initiate an intravenous (IV) line, the nurse applies the tourniquet to select the site. When should the nurse release the tourniquet? 1. After cleaning the insertion site 2. As soon as the needle pierces the skin 3. When the needle enters the vein 4. After the device is secured with tape

3. When the needle enters the vein The tourniquet causes the vein to become distended and makes entry into the vein easier. The tourniquet should be removed when the needle enters the vein. The option of removing the tourniquet after cleaning the insertion site and the option of removing the tourniquet as soon as the needle pierces the skin do not assist the nurse in keeping the vein distended and visible for complete insertion. Keeping the tourniquet on until after the device is secured could cause damage by impairing circulation.

A client is diagnosed with varicose veins and the nurse teaches the client about the pathophysiology associated with this disorder. The client asks, "What can I do to help myself?" What should the nurse respond? 1. "Limit walking to as little as possible." 2. "Reduce fluid intake to 1 L of liquid a day." 3. "Apply moisturizing lotion on your legs several times a day." 4. "Put on compression hose before getting out of bed in the morning."

4. "Put on compression hose before getting out of bed in the morning." As valves become incompetent, they allow blood to pool in the veins, which increases hydrostatic pressure and leads to further valve destruction. Compression hose provides external pressure, thereby facilitating venous return minimizing blood pooling in the veins. The legs are less congested after sleeping and therefore the hose should be put on before getting out of bed in the morning and before the legs are in the dependent position. The client should engage in exercise such as walking or swimming because muscle contraction encourages venous return to the heart. Prolonged sitting, standing, or crossing the legs should be avoided because they reduce venous return. Limiting fluid intake will not alter the leakage of fluid or blood into the interstitial space; this occurs in response to the increased hydrostatic pressure in the veins. Although applying moisturizing lotion may make the skin more supple, it will not treat enlarged and tortuous veins.

A client weighing 125 kilograms (275 pounds), is considered to be in septic shock when the mean arterial pressure is less than 65 mm Hg or the systolic blood pressure is less than 90 mm Hg after receiving how many liters of intravenous crystalloids? 1. 0.5 L 2. 1 L 3. 2 L 4. 5L

4. 5L This question requires the learner to apply the formula for defining septic shock to the scenario described. A septic client is considered to be in septic shock if the client remains hypotensive, as defined in the stem of the question, in spite of receiving 20-40 mL/kg of crystalloids, making 5 L the correct option.

A client who has been diagnosed with acute lymphocytic leukemia will be receiving doxorubicin (Adriamycin) infusions as part of a chemotherapy regimen. The nurse monitors the client for signs and symptoms of doxorubicin toxicity. What clinical finding indicates that toxicity has occurred? 1. Alopecia 2. Dyspnea 3. Metallic taste to food 4. Abnormalities in cardiac rhythm

4. Abnormalities in cardiac rhythm Doxorubicin is cardiotoxic, which is manifested by transient ECG abnormalities. Alopecia is an expected side effect of doxorubicin, not a toxic effect. Dyspnea and a metallic taste to food are not effects of doxorubicin.

A health care provider prescribes enoxaprarin (Lovenox) 30 mg subcutaneously daily. To ensure client safety, which measure would the nurse take when administering this medication? 1. Remove air pocket from prepackaged syringe before administration. 2. Rub site after administration. 3. Push over 2 minutes. 4. Administer in the abdomen.

4. Administer in the abdomen. Enoxaprarin specifically targets blood clots throughout the body and carries a lower risk of hemorrhage than that associated with the drugs heparin and warfarin. Enoxaprarin is administered once a day through a subcutaneous injection site around the naval. Enoxaprarin should be injected into the fatty tissue only, which is why the abdomen is the recommended injection site. Avoid administering in a muscle. Manufacturer recommendations indicate the air pocket from prepackaged syringes not be removed before administration. Rubbing the site is contraindicated as it can cause bruising. There are no recommendations to push medication over 2 minutes.

The nurse understands that clients with emphysema experience which pathophysiological change in the lungs? Alveolar sacs: 1. Collapse 2. Retain CO2 3. Become fluid filled 4. Become over distended

4. Become over distended Clients with emphysema experience changes in the alveolar sacs when they lose elasticity. Trapped air causes the sacs to become distended and can cause them to rupture. This in turn impairs gas exchange. The alveolar sacs do not collapse; however, they can rupture. The sacs do not retain CO2 nor do they become fluid filled.

The nurse instructs the client admitted for an acute exacerbation of chronic obstructive pulmonary disease (COPD) about the importance of assessing for right sided heart failure after discharge. The nurse instructs the client to assess for: 1. Increased appetite 2. Clubbing of the nail beds 3. Hypertension 4. Weight gain

4. Weight gain The most common signs and symptoms of right sided heart failure are hepatomegaly, weight gain, jugular vein distention, and peripheral edema. Clients with right sided heart failure often have decreased appetites. Clubbing is indicative of hypoxemia. Hypertension is associated with left sided heart failure.

A child in sickle cell crisis is admitted to the pediatric unit. Supportive care includes: Select all that apply. 1 Bedrest 2 Strict isolation 3 Oxygen via nasal cannula 4 Acetylsalicylic acid (Aspirin) 5 Acetaminophen (Tylenol) 6 Age appropriate activities 7 Intravenous hydration

1 Bedrest 3 Oxygen via nasal cannula 5 Acetaminophen (Tylenol) 6 Age appropriate activities 7 Intravenous hydration Bed rest decreases oxygen consumption. Providing oxygen via nasal cannula provides additional oxygen, which is necessary because of decreased hemoglobin, which carries oxygen. Acetaminophen will control fever and help manage pain. Age appropriate activities can help alleviate boredom as the child begins to feel better. Providing intravenous hydration until the child is able to tolerate adequate by mouth fluids reduces sickle cell clotting. Strict isolation would not be necessary unless placed on neutropenic precautions. Aspirin should not be given to children because of risk of Reyes syndrome.

A client with chronic obstructive pulmonary disease will be taking long-term oral corticosteroid therapy. After the nurse conducts a teaching session, which statement by the client indicates that further education is needed? Select all that apply. 1. "My urine may become discolored." 2. "I need to avoid crowds in enclosed areas." 3. "I will lose weight while on this medication." 4. "The medication should be taken between meals." 5. "When I'm feeling better, I can stop taking this medication." 6. "I will not take aspirin or ibuprofen while on this medication."

1. "My urine may become discolored." 3. "I will lose weight while on this medication." 4. "The medication should be taken between meals." 5. "When I'm feeling better, I can stop taking this medication." Corticosteroids do not cause urine discoloration; they do cause weight gain. Corticosteroids must be taken with milk or food to reduce gastrointestinal upset. Corticosteroids must not be stopped abruptly because of the risk of acute adrenal insufficiency; the medication will be tapered before it is discontinued. Long-term use of corticosteroids results in depressed immunity and may cause a greater risk for infection; therefore it is important to avoid large crowds of people and those with infections. It is important to avoid taking aspirin and nonsteroidal antiinflammatory drugs (NSAIDs) such as ibuprofen because of the possibility of gastric irritation and possible gastric bleeding.

A client is on a cardiac monitor. The monitor begins to alarm showing ventricular tachycardia. What should the nurse do first? 1. Check for a pulse. 2. Start cardiac compressions. 3. Administer oxygen via an ambu bag. 4. Prepare to defibrillate the client.

1. Check for a pulse. The treatment of ventricular tachycardia depends on the presence of a pulse. Therefore, checking for a pulse is the first priority for the nurse. Cardiac compressions would not be initiated if there was a pulse. Administering oxygen via an ambu bag would only occur if the client was not breathing. The client is not automatically defibrillated. Cardioversion is recommended for slower ventricular tachycardia.

The nurse is monitoring a client who is having a third transfusion of packed red blood cells. Which of these may be evident if the client is experiencing a febrile transfusion reaction? Select all that apply. 1. Chills 2. Urticaria 3. Hypotension 4. Tachycardia 5. Bronchospasm 6. Sense of impending doom

1. Chills 3. Hypotension 4. Tachycardia Febrile transfusion reactions occur most often in patients who have had multiple transfusions. Symptoms include chills, hypotension, tachycardia, tachypnea, and fever. Urticaria and bronchospasm occur with allergic transfusion reactions. Feeling a sense of impending doom occurs with hemolytic transfusion reactions.

A female client whose ECG exhibits multiple premature ventricular complexes is to take oral disopyramide (Norpace). Which side effects should the nurse include when teaching the client about this drug? Select all that apply. 1. Dry mouth 2. Rhinorrhea 3. Constipation 4. Hyperglycemia 5. Stress incontinence

1. Dry mouth 3. Constipation Dry mouth occurs because of its anticholinergic properties. Constipation is a side effect of this nonnitrate antidysrhythmic because of its anticholinergic properties. A thin, watery discharge from nose (rhinorrhea) does not occur with this medication because of its anticholinergic properties. Hypoglycemia, not hyperglycemia, may occur. Urinary hesitancy and retention, rather than stress incontinence, occur.

A client develops subcutaneous emphysema after the surgical creation of a tracheostomy. What assessment by the nurse most readily detects this complication? 1. Palpating the neck or face 2. Evaluating the blood gases 3. Auscultating the lung fields 4. Reviewing the chest x-ray film

1. Palpating the neck or face Subcutaneous emphysema refers to the presence of air in the tissue that surrounds an opening in the normally closed respiratory tract; the tissue appears puffy, and a crackling sensation is detected when trapped air is compressed between the nurse's palpating fingertips and the client's tissue. Gas exchange and thus blood gases are not affected. The lungs are not affected.

A blood transfusion of packed cells has been prescribed for a client. The client shows signs of hemolytic reaction. Place the appropriate nursing actions in order. 1. Stop the transfusion 2. Notify health care provider and blood bank 3. Run 0.9 normal saline at rapid rate 4. Change IV administration set

1. Stop the transfusion 4. Change IV administration set 3. Run 0.9 normal saline at rapid rate 2. Notify health care provider and blood bank The priority is to stop the transfusion. Failure to do so will make the reaction worse. Changing the intravenous administration set will avoid infusing any blood product remaining in tubing. Running normal saline rapidly will help to decrease shock and hypotension. Notifying the health care provider and blood bank would be the last step as this would take longer than the first three choices.

To prepare a client for surgery, which explanation by a nurse would be accurate related to pneumatic compression devices? 1. They help the venous blood return to the heart. 2. They will not cause discomfort, but gently massage the legs. 3. They are used instead of anticoagulant therapy. 4. They must be worn until the first time the client gets out of bed.

1. They help the venous blood return to the heart. Deep vein thrombosis (DVT) is a potential complication of any surgery lasting longer than 30 minutes. The purpose of pneumatic compression devices is to increase venous return. Clients often complain about pneumatic compression devices being hot and itchy. In addition to the pneumatic compression devices, a mechanical form of DVT prophylaxis, pharmaceutical prophylaxis is often required. Pneumatic compression devices are continued until the client is up ambulating frequently throughout the day.

The primary health care provider has prescribed a stat chest x-ray and electrocardiogram for an 85-year-old client with a history of congestive heart failure. The pulse oximeter has changed from 90% to 86% oxygen saturation. The nurse's immediate actions include which of the following? Select all that apply. 1. Tell a staff member to get the electrocardiogram machine. 2. Notify the x-ray department that a chest x-ray must be done stat. 3. Increase the supplemental oxygen without a prescription from 2 L nasal cannula to 4 L nasal cannula and notify the health care provider. 4. Have a staff member notify the nursing supervisor of the change in client status. 5. Notify the health care provider of the change in the oxygen saturation to ask what to do. 6. Tell the certified nursing assistant to get a prescription from the health care provider to increase the oxygen.

1. Tell a staff member to get the electrocardiogram machine. 2. Notify the x-ray department that a chest x-ray must be done stat. 3. Increase the supplemental oxygen without a prescription from 2 L nasal cannula to 4 L nasal cannula and notify the health care provider. 4. Have a staff member notify the nursing supervisor of the change in client status. A staff member can get the electrocardiogram machine and start the procedure. Ancillary personnel are trained to do electrocardiograms even if they are not able to interpret the results. Anyone can notify the x-ray department that the chest x-ray must be done. It is important to delegate the tasks to a specific person. Increasing the oxygen without a prescription is appropriate in the short term, but the nurse must obtain a prescription when notifying the health care provider. Notifying the health care provider of the change in oxygen saturation is appropriate, but it would be expected that nursing judgment had taken place and the oxygen already was increased from 2 L/min. Telling the certified nursing assistant (CNA) to get a prescription is an inappropriate action as a CNA is not allowed to take medical prescriptions. Taking a medical prescription is a nursing role.

A blood transfusion of packed cells has been prescribed for a client. The transfusion started five minutes ago and the client is complaining of chest pain and nausea, having difficulty breathing, and chills. The blood pressure has dropped from 140/88 to 110/60 mm Hg, temperature is 99.2 degrees, and the client seems less alert. The nurse suspects: 1 Urticarial reaction. 2 Hemolytic reaction. 3 Circulatory overload. 4 Anaphylactic reaction.

2 Hemolytic reaction. Chest pain, nausea, having difficulty breathing, and chills are signs of hemolytic reaction, which occurs with incompatible blood. Later come symptoms of shock and loss of consciousness. This type of reaction occurs within minutes of starting the infusion. Urticarial reactions are minor allergic reactions that typically have hives. Circulatory overload typically would occur with rapid infusion and would raise the blood pressure. An anaphylactic reaction would cause respiratory or cardiac collapse.

A client has coronary artery bypass graft (CABG) surgery for the second time via a sternal incision. What should the nurse teach the client to expect when returning home? 1 No further drainage from the incisions 2 Increased edema in the leg that provided the donor graft 3 Mild incisional pain and tenderness for three to four weeks 4 Extreme fatigue and a mild fever occurring for several weeks

2 Increased edema in the leg that provided the donor graft Because the client is out of bed more at home and the leg used for the donor graft is in the dependent position, edema of this extremity usually increases. The internal mammary artery is the graft of choice and was probably used in the first CABG procedure, necessitating retrieval of a vessel from the leg. Serosanguinous drainage may persist after discharge. Mild incisional pain and tenderness may persist longer than 3 to 4 weeks because it takes 6 to 12 weeks for the sternum to heal. Extreme fatigue and a mild fever are not expected; these are associated with postpericardiotomy syndrome and should be reported to the health care provider immediately.

A client is seen in the clinic with sickle cell anemia. The hemoglobin range that is expected to be seen in this client in sickle cell crisis would be: 1 3--4g/100 mL 2 6--8g/100 mL 3 12--14g/100 mL 4 16--18g/100 mL

2. 6--8g/100 mL In sickle cell crisis, hemoglobin values are usually in the 6--8g/100 mL range showing many sickle shaped cells, and the client also will have a low oxygen level. A level of 3--4g/100 mL would not carry enough oxygen as hemoglobin carries oxygen. A range of 12--14g/100 mL is not indicative of anemia. 16--18g/100 mL may be indicative of dehydration rather than anemia. Normal range for hemoglobin is For men, 13.5 to 17.5 grams per deciliter. For women, 12.0 to 15.5 grams per deciliter.

A client with supraventricular tachycardia (SVT) is being treated with diltiazem hydrochloride (Cardizem). What assessment indicates to the nurse that the diltiazem hydrochloride is effective? 1. Blood pressure of 90/60 mm Hg 2. Heart rate of 110 beats per minute 3. No longer complaining of heart palpations 4. Increased urine output

2. Heart rate of 110 beats per minute Diltiazem hydrochloride's purpose is to slow the heart rate down. SVT has a heart rate of 150 to 250 beats per minute. A heart rate of 110 indicates that the diltiazem hydrochloride is having the desired effect. Hypotension is a side effect of diltiazem hydrochloride, not a desired effect. Heart palpations are experienced by some with various dysrhythmias. A decreased sensation of heart palpations is a positive finding but is not present in all clients. Increased urine output may occur over a period of time because of the increased ventricular filling time, but would not occur until after the heart rate had stabilized.

A client receiving hemodialysis has an external shunt for circulatory access. With which life-threatening complication associated with external cannulas should the nurse be most concerned? 1. Infection 2. Hemorrhage 3. Skin breakdown 4. Impaired circulation

2. Hemorrhage Exsanguination can occur in a matter of minutes if cannulas are dislodged. Infection, skin breakdown, and impaired circulation are not life-threatening situations; preventing exsanguination takes priority.

A nurse is caring for a client experiencing an acute episode of bronchial asthma. What should nursing interventions achieve? 1. Curing the condition permanently 2. Raising mucous secretions from the chest 3. Limiting pulmonary secretions by decreasing fluid intake 4. Convincing the client that the condition is emotionally based

2. Raising mucous secretions from the chest In addition to dilation of bronchi, treatment is aimed at expectoration of mucus. Mucus interferes with gas exchange in the lungs. Curing the condition permanently is an unrealistic goal; asthma is a chronic illness. Increased fluid intake helps liquefy secretions. Asthma has a psychogenic factor but this is not the only cause; it may occur as an allergic response to an antigen, such as dust.

A client is extubated in the postanesthesia care unit after surgery. For which common response should the nurse be alert when monitoring the client for acute respiratory distress? 1. Bradycardia 2. Restlessness 3. Constricted pupils 4. Clubbing of the fingers

2. Restlessness Inadequate oxygenation of the brain may produce restlessness or behavioral changes. The pulse increases with cerebral hypoxia. The pupils dilate with cerebral hypoxia. Clubbing of the fingers is the result of increased vascularization and reflects a response to prolonged hypoxia.

A child has been diagnosed with hemophilia type A after experiencing excessive bleeding from a minor trauma. The mother shares that she is four weeks pregnant and questions as to whether this pregnancy will result in a child with hemophilia. The best response is: 1. Probably not, as there is a 50% risk of a mother who is a carrier transmitting the disease, and one child already has the condition. 2. With each pregnancy, there is a 50% chance of a carrier transmitting the condition or being a carrier, depending on the sex of the child. 3. Definitely, because the one child is hemophiliac, all future pregnancies will result in children with the condition. 4. If both parents have the condition, the child automatically will have hemophilia.

2. With each pregnancy, there is a 50% chance of a carrier transmitting the condition or being a carrier, depending on the sex of the child. With each pregnancy there is a 50% chance of a carrier transmitting the condition or being a carrier. The odds are the same with each pregnancy and do not change based upon a previous pregnancy. One child currently having the condition does not affect this pregnancy. A recessive trait will not automatically mean all future pregnancies will result in children with hemophilia. Both parents having the condition will not result necessarily in a child having hemophilia, as this is a recessive trait.

A client is seen in the clinic with sickle cell anemia. A brief explanation for this condition is: 1. rod shaped frágil platelets 2. abnormally shaped red blood cells 3. abnormally shaped white blood cells 4. Anemia resulting from deficiency of vitamin B12

2. abnormally shaped red blood cells Sickle cell anemia is abnormally shaped red blood cells that form a rod shape. The disorder affects hemoglobin rather than platelets. White blood cells are connected with infection. Anemia caused by B12 deficiency is pernicious anemia.

When caring for a client who has hyponatremia, the nurse would monitor for which of the following? 1Increased urine output 2 Deep rapid respirations 3 Decreased specific gravity 4 Distended neck veins

3 Decreased specific gravity A normal sodium level is between 135 and 145 mEq/L of sodium. Hyponatremia occurs when the sodium level falls below 135 mEq/L. Decreased specific gravity may be caused from aldosteronism, excessive fluid intake, diabetes insipidus central, diabetes insipidus nephrogenic, kidney failure, renal tubular necrosis, or severe kidney infection (pyelonephritis). Symptoms are nonspecific and can include mental changes, headache, nausea and vomiting, tiredness, muscle spasms, and seizures. Increased urine output, deep rapid respirations, and distended neck veins are not associated with hyponatremia.

While receiving a blood transfusion, a client develops flank pain, chills, and fever. What type of transfusion reaction does the nurse conclude that the client probably is experiencing? 1 Allergic 2 Pyrogenic 3 Hemolytic 4 Anaphylactic

3 Hemolytic A hemolytic transfusion reaction results from a recipient's antibodies that are incompatible with transfused red blood cells; it is called a type II hypersensitivity. The clinical findings are a result of red blood cell hemolysis, agglutination, and capillary plugging. An allergic transfusion reaction is the result of an immune sensitivity to foreign serum protein; it is called a type I hypersensitivity and associated clinical findings include urticaria, wheezing, dyspnea, and shock. Bacterial pyrogens are present in contaminated blood and can cause a febrile transfusion reaction; associated clinical findings include fever and chills, but not flank pain. There is no transfusion reaction called an anaphylactic transfusion reaction; anaphylaxis may occur with an allergic transfusion reaction.

A terminally ill client in a hospice unit for several weeks is receiving a morphine drip. The dose is now above the typical recommended dosage. The client's spouse tells the nurse that the client is again uncomfortable and needs the morphine increased. The prescription states to titrate the morphine to comfort level. What should the nurse do? 1. Add a placebo to the morphine to appease the spouse. 2. Discuss with the spouse the risk for morphine addiction. 3. Assess the client's pain before increasing the dose of morphine. 4. Check the client's heart rate before increasing the morphine to the next level.

3. Assess the client's pain before increasing the dose of morphine. Over time clients receiving morphine develop tolerance and require increasing doses to relieve pain, thus requiring continuing reassessments. Adding a placebo to the morphine to appease the spouse will not meet the client's need for relief from pain. The client is terminal and the risk for addiction is of no concern. The respiratory, not heart, rate is the significant vital sign to be monitored; morphine depresses the central nervous system, specifically the respiratory center in the brain.

The health care provider prescribes aminophylline to be given intravenously for the client experiencing an acute asthma attack. The nurse should instruct the client that the purpose of this medication is to act as a(n): 1. Antibiotic. 2. Antihistamine. 3. Bronchodilator. 4. Expectorant.

3. Bronchodilator. Aminophillyine is a bronchodilator. It relaxes the smooth muscles in the bronchial airway and relieves bronchospasms. This in turn improves air exchange. An antibiotic is used to treat a bacterial infection. An antihistamine blocks the action of histamine. An expectorant is used to loosen mucus in the lungs. An antihistamine, an antibiotic, or an expectorant will not relax the smooth muscles in the bronchial airway for clients experiencing an acute episode.

A client is admitted to the hospital with multiple signs and symptoms associated with a cardiac problem. What clinical finding alerts the nurse that the health care provider probably will insert a pacemaker? 1. Angina 2. Chest pain 3. Heart block 4. Tachycardia

3. Heart block Heart block is the primary indication for a pacemaker because there is an interference with the electrical conduction of impulses from the atria to the ventricles of the heart. The primary treatment for angina is medication; angina is not an indication for a pacemaker. The primary treatment for chest pain is medication; chest pain is not an indication for a pacemaker. The primary treatment for tachycardia is medication; tachycardia is not an indication for a pacemaker.

What should a nurse do to decrease or control the sensory and cognitive disturbances that can occur after a client has open heart surgery? 1. Restrict family visits 2. Withhold analgesic medications 3. Plan for maximum periods of rest 4. Keep the room light on most of the time

3. Plan for maximum periods of rest Sleep deprivation alone can cause these disturbances because of the interruption in rapid eye movement (REM) sleep. Lack of contact with significant others increases anxiety and feelings of isolation, which can lead to disturbances in rest. Pain limits or interrupts periods of sleep and rest. Analgesics should be administered as prescribed. Constant light increases cerebral arousal and limits sleep.

A client being treated for uncontrolled hypertension and chest pain calls out to the nurse and reports he or she is having a nose bleed. Upon entry to the client's room, the nurse immediately applies pressure. Which action should the nurse take next? 1. add humidity to the client's oxygen prescribed at 2 L/minute via nasal cannula 2. assess the client for further injuries indicative of a possible fall 3. auscultate the client's blood pressure 4. assess the client's pulse rate

3. auscultate the client's blood pressure Nose bleeds in adults often are indicative of hypertension in adults. While oxygen can dry out the mucus membranes in the nose, and assessing the client for further injuries is plausible, the nurse's first action should be to assess blood pressure, especially because the client was admitted for uncontrolled hypertension.

The health care provider prescribes enalapril maleate (Vasotec). Which instruction should the nurse include when educating the client about the new medication? 1 Check your pulse before taking the medication. 2 Use requires weekly basic metabolic panels to be drawn. 3 Take the medication with orange juice. 4 Change positions slowly.

4 Change positions slowly. Enalapril is an angiotensin-converting enzyme inhibitor and can cause postural hypotension. For safety purposes, the client should be instructed to change positions slowly to avoid dizziness or fainting. Checking pulse rate is not indicated before administration; checking blood pressure is indicated. While electrolytes often are checked for clients with hypertension and receiving medication therapy, weekly basic metabolic panels are not required while taking this medication. It is not necessary to take the medication with orange juice.

A child has been diagnosed with hemophilia type A after experiencing excessive bleeding from a minor trauma. The parents ask how this could happen in addition to many other questions. Hemophilia A is linked to a deficiency in: 1 Factor II 2 Factor III 3 Factor IX 4 Factor VIII

4 Factor VIII Hemophilia type A, which is the most common type of hemophilia, is from a deficiency of Factor VIII. Factors II and III are distractors. Factor IX is associated with hemophilia type B.

Digoxin (Lanoxin) and furosemide (Lasix) are prescribed for a client with the diagnosis of pulmonary edema. What client response to digoxin is unrelated to toxicity? 1 Nausea 2 Yellow vision 3 Irregular pulse 4 Pulse of 64 beats/min

4 Pulse of 64 beats/min A pulse of 64 beats/min is acceptable when the client is receiving digoxin; digoxin lengthens the atrioventricular conduction time, which slows the heart rate; toxicity may be present if the heart rate drops to less than 60 beats/min. Nausea is a symptom of toxicity; nausea and vomiting can occur because of gastric irritation and its action at central nervous system sites. Yellow vision is a symptom of toxicity; xanthopsia (yellow vision) is caused by digoxin's effect on visual cones. An irregular pulse is a sign of toxicity; premature nodal or ventricular impulses and varying degrees of heart block can occur because of slowed transmission of impulses through the atrioventricular node.

The health care provider prescribes aminophylline 500 mg in 500 mL normal saline (NS) to infuse over 4 hours for a client who has been admitted for an acute exacerbation of chronic obstructive pulmonary disease (COPD). The tubing delivers 60gtt/min. The nurse should set the intravenous (IV) pump to administer how many milliliters per hour? 1. 60 mL/hr 2. 65 mL/hr 3. 120 mL/hr 4. 125 mL/hr

4. 125 mL/hr

A client who has a history of several myocardial infarctions is admitted to the hospital for an unrelated medical condition. Because of the client's history, the nurse is concerned about the possibility of the client experiencing right ventricular failure. For what early common indication of right ventricular failure should the nurse monitor the client? 1. Chest pain 2. Bradypnea 3. Bradycardia 4. Peripheral edema

4. Peripheral edema Increased venous pressure resulting from backup of blood, as the right ventricle of the heart fails, forces capillary fluid to seep into interstitial spaces, resulting in peripheral edema. Chest pain may be present with a myocardial infarction or cardiovascular insufficiency, not heart failure. Tachypnea and dyspnea, not bradypnea, occur with right ventricular failure. Bradycardia does not occur; the heartbeats may vary in intensity, a pulse deficit may be present, or a bounding pulse may be felt.

A client with a history of heart disease has been receiving a calcium channel blocker and morphine sulfate for pain from abdominal surgery. When getting the client out of bed, the nurse first should have the client sit on the edge of the bed with feet on the floor. What untoward client response can be prevented by this nursing action? 1. Abdominal pain 2. Respiratory distress 3. Sudden hemorrhage 4. Postural hypotension

4. Postural hypotension After administration of certain antihypertensives or opioids, a client's neurocirculatory reflexes may have some difficulty adjusting to the force of gravity when an upright position is assumed. Postural or orthostatic hypotension occurs, and blood supply to the brain is temporarily decreased. Abdominal pain, respiratory distress, and sudden hemorrhage will not be prevented by the intervention described.

A client is seen in the clinic with sickle cell anemia. The parents of the client ask how their child got sickle cell anemia. An explanation is: 1. Sickle cell anemia is a random condition with no known cause 2. if one parent is a carrier and one is negative for the gene , the child will get the disease 3. if both parents are carriers, all of their offspring will probably get this disease and they should consider sterilization 4. if both parents are carriers, the odds are one in four that an offspring will get the disease, and one in four that an offspring will be disease free

4. if both parents are carriers, the odds are one in four that an offspring will get the disease, and one in four that an offspring will be disease free If both parents are carriers, the odds are one in four an offspring will get the disease, two will be carriers, and one in four will be disease free. It is an autosomal recessive inherited condition. Sickle cell anemia is not random in that the gene must come from both parents. One parent being a carrier and the other not having the gene would not cause the disease. If both parents are carriers there is the possibility of 25% with each pregnancy that the child may inherit the disease, and 50% of being a carrier.

What is Hemophilia?

A disorder in which blood doesn't clot normally.

What are analgesics for?

Used to relieve pain

What is emphysema?

a condition in which the air sacs of the lungs are damaged and enlarged, causing breathlessness.

What is Petechiae

a small red or purple spot caused by bleeding into the skin.

What is hemophilia

medical condition in which the ability of the blood to clot is severely reduced, causing the sufferer to bleed severely from even a slight injury. The condition is typically caused by a hereditary lack of a coagulation factor, most often factor VIII.

What is Pneumothorax?

the presence of air or gas in the cavity between the lungs and the chest wall, causing collapse of the lung.


Kaugnay na mga set ng pag-aaral

Ch 36: Management of Patients with Musculoskeletal Disorders

View Set

Theology of Christian Marriage Final

View Set

Chapter 12 Practice Quiz [EXTRA]

View Set

PE Praxis 5091 Content and Knowledge, Form 2 Practice Test

View Set

Chapter 7 international business

View Set

ECO/365T: Principles Of Microeconomics - Week 3

View Set

13.3.5 Client Pro Practice Questions

View Set